¿Cómo debo planificar mi preparación para el GRE?

Whee! GRE es el único examen para el que me divertí mucho preparándome.

Un año es mucho tiempo, por lo que es bueno hacerlo. La forma en que debe prepararse para el GRE depende en gran medida de su situación actual.

La parte del examen de Razonamiento Cuantitativo (Matemáticas) es bastante fácil para cualquier persona con experiencia en ingeniería, por lo que se puede resolver resolviendo algunas pruebas de práctica para garantizar que su velocidad y precisión sean adecuadas. Apunta a que TODAS las preguntas sean correctas.

La parte del examen de razonamiento verbal es donde la mayoría de los estudiantes indios necesitan mucha preparación. Es importante entender aquí que la parte VR no tiene la intención de probar solo su vocabulario, per se. Más bien, tiene la intención de poner a prueba sus habilidades analíticas y de razonamiento, y el vocabulario es solo una herramienta. A través de ejercicios de comprensión y otras preguntas, el objetivo es ver qué tan bien puede analizar los argumentos y sacar conclusiones de un texto dado.

Vocabulario

  • Intente y calibre su vocabulario intentando algunas pruebas dadas en el libro Barrons GRE. Si crees que es decente (sabes al menos el 50% de las palabras), tu trabajo es bastante fácil. En mi caso, podría intentar alrededor del 65%.
  • Si crees que falta, hay un libro rojo (ahora es blanco) llamado Word Power Made Easy de Norman Lewis que muchos estudiantes parecen amar. Solo lo he leído ocasionalmente, por lo que no tengo una opinión firme al respecto, pero su enfoque etimológico es útil, especialmente si nunca ha visto el lenguaje de esa manera. Es bueno para el vocabulario, pero es mejor que ignores sus consejos gramaticales.
  • Intenta al menos completar las listas de palabras dadas en Barrons. No es exhaustivo, pero es una colección bastante buena. Hay alrededor de 3500 palabras ahí, y te llevará bastante tiempo si tu vocabulario no es bueno para empezar. Espere pasar alrededor de 3-4 horas por lista de palabras.
  • Aparte de esto, también puede consultar los recursos en línea

Razonamiento

  • Esto vendrá en gran parte con la práctica. A medida que trabaja en el vocabulario, siga resolviendo pruebas de práctica y analice cuántos de sus errores no se debieron a problemas de vocabulario, sino a problemas de lógica / razonamiento. Estos son los que necesitas para trabajar. Mejorará con la práctica y también mejorará su vocabulario al mismo tiempo.
  • Como dije antes, el GRE no es una prueba de vocabulario en bruto. En lugar de limitarse a asaltar palabras, trate de ponerlas en contexto. Tenga una idea de cómo se usan esas palabras, donde se usan esas palabras. Tener una idea de si una palabra se utiliza en un sentido de aprobación o un sentido de desaprobación. Trate de apreciar los matices entre las palabras que tienen un significado muy similar pero que aún no son intercambiables.

    Esto es lo que te ayudará más, tanto en la vida como en el examen.

Notas de Cierre

  • Si te preparas bien y te preparas adecuadamente, obtendrás un vocabulario y una capacidad de razonamiento mucho más sólidos. Este es un activo útil en el mundo académico, así como en la vida cotidiana.
  • Los estudiantes que comienzan con un vocabulario relativamente bajo y luego aumentan significativamente durante la preparación GRE a menudo sufren de lo que yo llamo síndrome de uso desagradable de palabras, donde un intento demasiado entusiasta de usar el vocabulario recién adquirido resulta en una prosa que no solo es patética sino Eso a menudo tiene poco sentido. Esto puede remediarse fácilmente si dedica tiempo a comprender exactamente cómo / dónde se usa una palabra en particular (como se mencionó anteriormente)

Tras otro envío de mi otra respuesta: la respuesta de Austin Li a ¿Cuál es la mejor manera de prepararse para un examen competitivo (CAT / GRE / GMAT) en 1 año sin pagar ninguna matrícula?

La guía rápida y sucia para el GRE: ¡Mejora tu puntuación de 10 a 20 en una semana!

Nota: Estoy escribiendo esto porque mi novia podría estar tomando el GRE en unos meses y quería escribir una guía para ella. Soy un idiota perezoso. No me gusta estudiar, y a pesar de comprar el Barron’s, el Kaplan y la guía oficial, solo estudié alrededor del 2% del contenido en una semana. Comencé con la prueba de diagnóstico y obtuve 160 cuantitativas, 150 verbales. En la prueba real obtuve 164 cuantitativos, 163 verbales y 5.5 AWA. El AWA es una broma. Sigue leyendo y te mostraré cómo hacerlo.

Si está leyendo esto y tomando el examen en una semana, la mayoría de estos consejos aún lo ayudarán.

  1. Esta es probablemente la estrategia más útil. Mantiene el control de tu tiempo y te libera del estrés de recordar tus pensamientos cuando la fatiga mental comienza a afectarte. Antes de que comience la prueba y entre las secciones de prueba, se le dan unos minutos de tiempo. Usa todo este tiempo para prepararte. Coloque el papel de desecho en el paisaje y use un bolígrafo para hacer una cuadrícula lógica deductiva. No se moleste en que una regla haga líneas bonitas, solo asegúrese de hacerlo y etiquete los números de las preguntas de una manera fácil de leer.

    Los problemas suelen tener de 4 a 5 opciones de opción múltiple. Si no está seguro de la respuesta, generalmente podrá decir que 2 a 3 opciones están equivocadas si conoce las palabras de matemáticas y vocabulario, y un resto de 2 opciones sobre las que no está seguro / olvidó el significado / No pude averiguar los pasos finales de las matemáticas.

    Cuadrícula: tenga en cuenta que en la sección verbal, no complete el número de la pregunta, ya que habrá algunas preguntas de varios espacios en blanco que deberá completar correctamente para obtener crédito (¡Asesino!) , Así que etiquete esas preguntas “1 a, 1 b , 1 c “.


    Tómese a lo sumo aproximadamente un minuto para cada pregunta. Desde la nueva revisión, es menos importante asegurarse de que responde correctamente a las primeras preguntas. Si ve una respuesta incorrecta, marque una “X” en la cuadrícula para esa opción. Si ve una respuesta de la que no está seguro entre dos, marque una “O” en la cuadrícula.

    En la sección verbal donde una pregunta tiene varios espacios en blanco para completar, probablemente terminará con una pregunta de la que no está seguro en casi todas las opciones, de las cuales necesita encadenar las “O” para ver cuál toma sentido. Solo hay 6 combinaciones en 3 espacios en blanco, cada uno con 2 opciones inseguras. Elimine de ellos y terminará con 2 a 3 opciones, y aumentará su probabilidad de éxito de 1/27 (3 opciones en 3 espacios en blanco) a aproximadamente 1 / 2.5 (2 opciones en 3 espacios en blanco, con 3-4 de 6 combinaciones eliminadas)

    Encierra en un círculo los números de pregunta de los que no estás seguro. Cuando haya terminado con la sección, puede retroceder rápidamente e identificar la pregunta que necesita revisar y rehacer.

    SIEMPRE HAGA UNA RESPUESTA. ¡Una conjetura todavía cuenta para los puntos!

  2. Compra las flash cards y duerme con ellas. Si ya los has comprado pero no te estás acostando con ellos, empieza a hacerlo ahora. Llévalos contigo a la cena, a los juegos, a la socialización y así sucesivamente. Rompe la pila de tarjetas de memoria flash en bucles de 50 tarjetas. Estudia a través de un bucle cada día, y pasa al siguiente. En los dos días posteriores a la finalización de un bucle, realice una prueba escrita con el bucle de la tarjeta flash que solo muestre la palabra pero no la definición. En siete días habrás memorizado 250 palabras para el GRE. Siempre elija de las tarjetas de una lista “más frecuente” para cubrir palabras de alta probabilidad.
  3. El AWA es una broma. Mucha gente pierde puntos innecesarios en él debido a las prácticas culturales y debido a la personalidad de la escritura que presentan que no les gustará a todos los evaluadores de exámenes. Es un ser humano que lee su ensayo, no un robot, no compre el argumento de que no cometen errores y no juzgue su estilo de escritura personal.

    ANALIZAR UN PROBLEMA

    Primero, eche un vistazo al resumen “Analizar un problema” del sitio web:

    • Escriba una respuesta en la que discuta en qué medida está de acuerdo o en desacuerdo con la declaración y explique su razonamiento para la posición que toma. Al desarrollar y respaldar su posición, debe considerar las formas en que la declaración podría o no ser cierta y explicar cómo estas consideraciones dan forma a su posición.
    • Escriba una respuesta en la que discuta en qué medida está de acuerdo o en desacuerdo con la recomendación y explique su razonamiento para la posición que toma. Al desarrollar y respaldar su posición, describa las circunstancias específicas en las que la adopción de la recomendación sería o no ventajosa y explique cómo estos ejemplos dan forma a su posición.
    • Escriba una respuesta en la que discuta en qué medida está de acuerdo o en desacuerdo con la reclamación . Al desarrollar y respaldar su posición, asegúrese de abordar los motivos y / o ejemplos más convincentes que podrían usarse para desafiar su posición.
    • Escriba una respuesta en la que discuta qué vista se alinee más estrechamente con su propia posición y explique su razonamiento para la posición que toma . Al desarrollar y respaldar su posición, debe abordar las dos opiniones presentadas.
    • Escriba una respuesta en la que discuta en qué medida está de acuerdo o en desacuerdo con la reclamación y la razón en que se basa esa reclamación. Escriba una respuesta en la que discuta sus puntos de vista sobre la política y explique su razonamiento para la posición que toma. Al desarrollar y respaldar su posición, debe considerar las posibles consecuencias de implementar la política y explicar cómo estas consecuencias dan forma a su posición.
  4. Observe cómo todos ellos señalan si está de acuerdo o en desacuerdo , y por qué. Su primera oración en esta sección siempre debe indicar en qué medida está de acuerdo o en desacuerdo. La segunda oración siempre debe resaltar las razones principales por las que está de acuerdo o en desacuerdo, e idealmente debería estar en coma en serie.

    “… de acuerdo / en desacuerdo debido a AAA, BBB y CCC”

    De esos puntos, tienes tus párrafos. Cada párrafo debe comenzar con la definición de su argumento exacto, y debe ir seguido de una segunda oración que explique cómo su argumento conecta / mejora / debilita el problema en cuestión.

    Elija los argumentos que se construyen unos sobre otros y en el párrafo de resumen, resúmalos y vuelva a exponer brevemente su posición.

    ANALIZAR UN ARGUMENTO

    Echa un vistazo a la respuesta modelo en el sitio oficial:
    Analizar un argumento

    Cuando mira la respuesta del modelo, aquí están los resultados que da a estas tres preguntas importantes:

  1. ¿La reclamación es válida o no? Sí, puede ser un reclamo válido para aumentar la financiación. [Tenga en cuenta que es muy importante separar la validez de la reclamación y la validez del razonamiento detrás de la reclamación.]
  2. ¿El razonamiento detrás de la reclamación es válido o no? No, la encuesta puede estar equivocada en su alcance, validez y relevancia. [Casi siempre hay un dato numérico en el pasaje que podría ser atacado o podría tener sus supuestos señalados.]
  3. ¿Hay problemas en la estructura lógica que socavan el argumento, pero no la afirmación? Sí, la causa del olor y la baja calidad del agua no se ha establecido para poder ser “limpiadas” de manera efectiva, y no se ha establecido un vínculo causal entre la calidad del agua y el uso. [Este tipo de estructura lógica suele ser lógica de pendiente resbaladiza. Lee abajo.]
  • Tenga en cuenta que cuanto más se desvíe de la evidencia en cuestión y haga girar sus propias teorías, más suposiciones hará y más débil se volverá su argumento. Si observas los ensayos de puntuación 5 y puntuación 4, sus ataques lógicos están menos explicados y menos basados ​​en la evidencia y la estructura del argumento.

    Preste atención a las estructuras de pendiente resbaladiza como el ejemplo. Una pendiente resbaladiza generalmente se basa en una larga cadena de supuestos aparentemente razonables para llegar a argumentos irrazonables o mal apoyados. Tome este ejemplo de una mamá que advierte a un niño que no vaya a una fiesta, porque una fiesta puede tener niños malos, que podrían hacerse amigos del niño en cuestión, y con el tiempo podrían introducir al niño a las drogas o al alcohol a medida que crecen, lo que podría desarrollarse. en una adicción, que podría arruinar las calificaciones de la escuela secundaria, destruir las posibilidades de admisión a la universidad y, en última instancia, no conseguir un trabajo bien pagado, y luego arruinar la vida del niño. De cada salto, el salto lógico puede parecer pequeño, pero al final el argumento se va a ir a la fiesta = la vida arruinada. Atacando la validez estadística, la conexión lógica entre un argumento y otro, las suposiciones hechas sobre la evidencia que determina si es circunstancial o irrefutable se convierten en los puntos principales del ensayo. Evite las ganas de impresionar a los escritores utilizando palabras extravagantes y estructuras gramaticales complejas. Use el principio KISS y escriba en una redacción sencilla para permitir que su argumento salga con un tono fuerte.

  • He estado enseñando el GRE por varios años. Lo que sigue está inspirado en las muchas sesiones que he tenido con los estudiantes, muchos de los cuales han ido muy bien en el GRE. Mucho de esto también es específico del GRE revisado, que hizo su debut en agosto de 2011.

    1. Usar material oficial.

    Al prepararse para el GRE, no hay preguntas que lo preparen mejor que las escritas por ETS, que es responsable de escribir las preguntas que verá el día del examen. En la medida de lo posible, aténgase a las preguntas de la Guía oficial del GRE para que pueda comprender cómo los escritores de pruebas quieren que usted razona.

    Puede que no creas que eso haga una gran diferencia, pero muchos editores tienen preguntas que requieren que hagas ciertos saltos de lógica, o emplees una lógica cuestionable, que nunca tendrás que hacer en la prueba real. Luego está el hecho de que muchas preguntas simplemente no son tan difíciles o matizadas como las preguntas que verás el día del examen. Este es un problema al que se enfrentan muchos estudiantes cuando usan Kaplan. Básicamente, está tomando una versión simplificada del GRE, con preguntas a menudo mal construidas.


    2. Tomar exámenes de práctica.

    No hay mejor manera de prepararse para el día de la prueba que tomar una prueba simulada. Te entrenarás para sentarte por más de tres horas mirando una pantalla, ya que eres golpeado por una avalancha de problemas GRE.

    Si es posible, use solo las pruebas oficiales, que se encuentran en la Guía oficial del nuevo GRE. Hay cuatro pruebas de este tipo: dos que vienen con el libro y dos que están basadas en computadora y vienen con el CD que forma parte del libro. Escurre esto durante las cuatro semanas que está preparando, de modo que no haya menos de cinco días entre las pruebas y no más de diez días. Si está estudiando por más tiempo, es posible que también desee considerar las pruebas GRE de Manhattan (ver más abajo).

    Finalmente, las pruebas de práctica te permitirán evaluar mejor tus fortalezas y debilidades, y te ayudarán a informar tus sesiones de estudio entre las pruebas.

    3. Centrarse en las debilidades

    Asegúrese de no dejarse llevar por las áreas en las que ya se destaca. Por ejemplo, si generalmente tiene una gran capacidad para resolver problemas de matemáticas, pero si no le gusta la interpretación de datos (esa es la sección con todos los gráficos), lo último que quiere hacer es evitar esa sección. En su lugar, conviértase en una prioridad para ser seguro y efectivo en todas las preguntas relacionadas con la Interpretación de datos.

    4. Usa el mejor material.

    Si bien el material oficial es claramente la mejor preparación que existe, no necesariamente proporciona las estrategias más efectivas. Es posible que desee echar un vistazo a la Revisión de Princeton para ver estrategias útiles en general. En cuanto a las estrategias específicas para casi todos los aspectos del GRE, desde las permutaciones hasta los argumentos de párrafos complicados, los libros de Manhattan GRE (MGRE) son los mejores. MGRE también tiene seis pruebas de práctica que imitan la prueba real muy bien (aunque esto es más con las matemáticas que con las verbales). Para más práctica en el formato en línea, Magoosh también ofrece preguntas que son similares, y con frecuencia más difíciles, que las que se encuentran en la prueba real. (Probablemente debería mencionar que trabajo para Magoosh :))

    Por supuesto, hay mucho material por ahí que debe evitarse, especialmente en términos de contenido de práctica. Aquí hay un desglose útil: 2013 Reseñas de libros GRE

    Hay algunos principios básicos para las personas que están tratando de prepararse por su cuenta. La elaboración de la técnica de estudio es algo más táctico y personal, que tendrás que hacer por tu cuenta, según lo que sabes sobre la mejor manera de aprender. La mayoría de estos, aparte de las distinciones indicadas, se aplican a las pruebas estandarizadas en general:

    1. Comience por crear un horario realista de trabajo diario . Debido a que la parte diaria es importante, lo realista probablemente se traduce en 15-20 minutos al día. “Voy a pasar dos horas por noche” tiene buenas intenciones, pero es poco probable que suceda.
    2. Usa solo los materiales oficiales . Solo hay 1 libro publicado para el GRE.
    3. Los materiales incluyen problemas de práctica y exámenes de práctica. Primero haga los problemas de la práctica, luego haga las pruebas de práctica regulares a medida que se acerca la fecha real de la prueba. Hay dos pruebas oficiales de práctica GRE que se pueden descargar gratis. Debido a que las pruebas son adaptativas, se pueden tomar varias veces. Puede salirse con alrededor de tres sesiones por prueba sin superposición significativa, por lo que le permite practicar seis pruebas. Estos deben tomarse una vez a la semana durante las seis semanas previas a la prueba, en un entorno que simule el entorno de prueba real lo más cerca posible (por ejemplo, levántese y comience la prueba en los momentos en que lo necesite, aislese en una habitación libre de distracciones, no te dejes escuchar música, coloca tu teléfono en otro lugar).
    4. Encuentra a alguien con quien puedas revisar las preguntas que te hayan equivocado . Esto no tiene que ser un profesional si no es factible para usted, pero debe ser alguien que sea bueno en el contenido relevante o que haya obtenido una buena calificación en la prueba anteriormente, o ambas cosas.
    5. Haga más problemas de práctica para las áreas de contenido que parecen estar surgiendo la mayor frecuencia de respuestas incorrectas en sus exámenes de práctica.
    6. Aunque no hay una lista de vocabulario oficial, la preparación GRE requiere estudiar vocabulario . Los antónimos y las analogías (y, en menor medida, las terminaciones de oraciones) se basan mucho más en el vocabulario que en el pasaje o vocabulario en contexto que las ha reemplazado en gran medida en exámenes como el SAT por esa misma razón (vea ¿Cómo han cambiado los SAT? en los últimos 20-30 años?). Se acercan mucho más a ser “o lo sabes o no”. Afortunadamente, hay una gran cantidad de libros de vocabulario por ahí, y solo hacer un poco cada día aumenta las probabilidades de que estas preguntas sean correctas.
    7. Finalmente, solo para subrayar , la preparación consistente y bien planificada es clave . En pruebas como el GRE y el GMAT, el cramming es básicamente un crapshoot gigante.

    A aquellos

    1. Quienes no tienen mucho tiempo para prepararse, como aquellos que tienen un trabajo (mi caso) o demasiado trabajo universitario.
    2. Preparándose por su cuenta, sin ningún tipo de coaching. Sin libros duros.
    3. Que son perezosos (Menos eufemísticamente)

    Aquí está mi breve:
    (Es una lectura bastante larga – TL; DR al final)

    No pude hacer lo que ellos hacen, debido a la falta de tiempo. Estas cosas no deberían decepcionarte.

    • “No hay tiempo para pruebas simuladas” : no tomé ninguna (de las 3) pruebas simuladas por completo; solo solía intentar las dos secciones de Quant, revisé los errores y, finalmente, terminé preparándome para Quant hasta el final del día. Intenté Verbal solo una vez (eso también mientras hacía triple clic y miraba furtivamente el significado de cada una de las palabras que me eran desconocidas) y obtuve solo 20 de 40 respuestas correctas. Esto fue antes de que comenzara los preparativos, así que no me importó mucho.
    • ÚLTIMOS MINUTOS Preparativos : A pesar de que todos me dijeron lo contrario, no podía salir de casa por la mañana sin desplazarme por mi lista de palabras (en su mayoría desprevenidas) y las notas de los principales errores cuánticos.
    • “Dormir lo suficiente antes del día” : ¡No tenía! 🙁 ¡Se siente mal preparado, solo se retuerce en la cama durante horas sin dormir!

    Lo que puedo hacer ahora

    Toda esa ansiedad de último minuto, la pereza hacia Verbal, la última noche de insomnio, fueron inmateriales. Lo más importante es estar tranquilo y concentrado durante el examen. ¡Eso es todo! 🙂

    Tu puntuación ya está decidida; Una preparación de unos pocos meses sería insignificante frente a todas las Matemáticas / Inglés que ha adquirido durante los últimos 20 a 25 años de su vida.

    El post de Ananth Balasubramanian es el más verdadero. Un sincero agradecimiento a él. 🙂
    [Aquí está el enlace: GRE breve por Ananth Balasubramanian | Facebook]

    Material de preparación sugerido (es decir, los que he mencionado)

    • Manhattan 5 lb. (el favorito favorito) para Quant. Siendo un amante de las matemáticas, resolví todos los problemas y pruebas en esto.
    • Guía oficial de ETS – devorarla completamente! ¡Eructar!
    • ETS Quant Practice y ETS Verbal Practice libros: los mejores para entender el tipo y la variedad de preguntas formuladas y para desarrollar sus propias estrategias.
    • Aplicación Magoosh Vocabulary Builder : para los momentos en que solía tomarme un descanso o para ir al trabajo o para pasar el rato antes de irme a dormir.

    Consejos cuantitativos

    • Practica todo tipo de preguntas de todos los temas. Ni siquiera ignore las preguntas de álgebra simples como “1 + 3 =?”, Porque debe haber un truco involucrado. Incluso una vez lo haría, si agarra las cosas rápidamente.
    • Luego revise las respuestas y siga anotando los errores que comete. Estos deben ser útiles en los preparativos de ÚLTIMO MINUTO. (Por ejemplo, no ignore las fracciones y los valores negativos en x2; revise las preguntas de DI; esa nueva fórmula de ‘muestra de desviación estándar’, etc.) Si tiene tiempo, revise las respuestas a las correctas también. Puede haber También se proporcionó algo de sabiduría, por ejemplo, en los libros de Manhattan y ETS.

    Consejos verbales

    • Practica RC tanto como puedas. No es importante que termines todo el libro de RC99. Practiqué solo unos 15-20 ensayos de los libros que he mencionado. Lo importante es aprender a pensar críticamente sobre las preguntas y las opciones. De vez en cuando, tendrías que razonar con las opciones como: “¡Hey! El pasaje dice que las manzanas son malas. Pero no dice “específicamente” que son malos en sabor. Así que elimina ”. El razonamiento se vuelve aún más estricto que esto. Aprende a hacerlo, y RC es tu juguete.
    • Además, la lectura de velocidad en RC ayuda mucho. Las secciones verbales pueden ser tomadas por usted.
    • SE y TC – Ve con tu instinto. Eliminar. La práctica no me ayudó mucho, supongo. Pero para tener confianza en el examen, practica. 🙂

    Consejos de AWA

    No creo que sea la persona adecuada para hablar de ello. Solo leí algunas muestras del ETS OG, el día anterior al día D Me gusta escribir, así que también podría escribir mucho en el examen.

    • Todo esto lo hice dentro de 10 días efectivamente. La falta de sueño no me molestó en el día del examen. (Podría molestar a los demás, ¡duerme!)
    • Sin embargo, durante los preparativos, creo que el sueño es importante, ya que lo que aprendas necesita el calor de la almohada para estar feliz de permanecer en tu cerebro …
    • Tan importantes son las tareas cotidianas y el juego: solía jugar al menos dos juegos de Carrom todos los días durante mi preparación. Pero NO hagas nada que te haga sentir culpable por haber perdido el tiempo.

    El día de la prueba

    • La noche anterior había dormido solo durante 4 horas, se despertó a las 6:30 AM, solo comió un plátano para el desayuno y llegó al lugar a las 7:30 AM. Para mi sorpresa, el control de seguridad ya había comenzado! : O Además, para aumentar el tormento, ¡el ‘Prometric Intro video’ se estaba reproduciendo en el televisor allí en bucle! Grrr!
    • Los temas de AWA eran buenos, tenían amplios pensamientos para escribir. El tiempo también es suficiente para que usted escriba 400-500 palabras.
    • ¡Feliz de ver la sección de Quant primero! 😀 Tengo todas las respuestas. Sin embargo, era más complicado y más largo que la segunda sección de Quant, por lo que sabía que era experimental.
    • Y la mayoría de los pasajes verbales que recibí estaban relacionados con la ciencia: astronomía, química, botánica, etc. Suerte, porque me encanta leer sobre ellos. 🙂
    • A Little Time Hack : No presione el botón Continuar cuando termine la sección. Respira profundamente. Prepare su mente para cambiar de Q a V o V a Q. Luego presione Continuar y disfrute de una pausa oficial de 60 segundos más … 😛 (Había leído esto en algún lugar, así que implementado).
    • Tiempo de descanso: Snickers. No es de extrañar. (Creo que ETS debería escribir esto en el PDF de las convenciones GRE ahora)
    • Todo salió bien. Y la pantalla de puntuación. Mis ojos fueron a la sección Cuantitativa primero: 170. Sin emoción. Multa. Esperado. Luego pasaron a la sección verbal: 159. Lo agregué a mi puntaje de Quant y … “¡WTF! Se está convirtiendo en 329! Quizás mi cerebro está demasiado cansado para hacer una simple adición ahora “, me dije a mí mismo. Hice la matemática otra vez. Y luego otra vez en la hoja áspera amarilla – ¡ahí es cuando se estableció!

    Sonreí, llené las universidades y fui directamente a mis amigos que esperaban una fiesta. 🙂

    TL; DR

    CONOCER los tipos de preguntas; PRACTICAR hasta que empieces a obtener las respuestas correctas; CHILL a menudo: lo más importante, durante el examen.

    ¡Todo lo mejor! 🙂

    [Si alguien tiene más preguntas, estaré encantado de ayudarle. 🙂]

    Internet está inundado de foros y grupos increíbles para los aspirantes a GRE. En la era de la educación en línea, las ventajas de la interacción uno a uno, la capacitación en línea personalizada, la eliminación de dudas y muchos otros aspectos del aprendizaje son lo que debe aprovechar para darle una ventaja a su preparación.

    Sin embargo, me gustaría enfatizar aquí que a pesar de que los foros son excelentes fuentes de información, uno no podrá lograr calificaciones altas usando solo foros y grupos de discusión. ¡No es una idea prudente! Uno requiere una guía experta para obtener un puntaje alto y aterrizar en la universidad de sus sueños.

    Dicho esto, aquí hay una lista de foros que pueden resultarle útiles:

    • Urch-GRE: Urch es un gran lugar para que los estudiantes interactúen con otros aspirantes a GRE. Todavía tiene algunos inconvenientes. Encuentre los pros y los contras específicos del foro de Urch-GRE en esta Tabla.

    • GradCafe: los temas en este foro son amplios, desde el análisis de AWA hasta los preparativos de Visa. Este foro difiere de Urch y también tiene una serie de pros y contras:

    • Foro del Dr. Raju: Este es otro buen foro para los preparativos verbales y cuantitativos de GRE. Hay temas que abordan temas relacionados con Visa y otros requisitos de admisión.

    • GRE PrepClub-Forum: este sitio web está dedicado a GRE y tiene foros detallados de GRE. Los foros ofrecen un análisis en profundidad y detallado de cada tema.

    • Edulix forum: es conocido por su sub-foro de evaluación de perfiles. También es un foro de preparación GRE bien pensado, todavía carece del volumen de hilos de preparación GRE.

    Para obtener detalles sobre otros foros, puede consultar este blog: Los mejores foros en línea para la preparación de GRE.
    Hay algunos grupos de Facebook que pueden ser útiles para un estudiante que se prepara a sí mismo y que busca respuestas rápidas a sus dudas. Mira estos:

    1. MS en EE. UU. Otoño ‘2016: Aunque parece un poco anticuado, el grupo está activo. Las personas regularmente publican sus dudas y obtienen respuestas de sus compañeros aspirantes y otras personas experimentadas. Es una gran comunidad de personas con ideas afines que buscan el éxito en GRE.
    2. MS Dreams: este es otro grupo con más de 6 k miembros que buscan aterrizar en las universidades de sus sueños para obtener una maestría. La ventaja añadida en este grupo es que a medida que los aspirantes interactúan entre sí, los expertos de GREedge responden a cada duda con un análisis cuidadoso de la duda. El asesoramiento de expertos es una gran ventaja cualquier día. El grupo también realiza pruebas periódicas y otras actividades para mantener a los aspirantes bien informados y comprometidos.

    Para resumir hay algunas cosas buenas y algunas no tan buenas de los grupos de usuarios generales y foros. Para indicar algunos:

    Por lo general, las respuestas no provienen de expertos, por eso las respuestas no se curan ni se examinan cuidadosamente. Todo tipo de comercializadores y promotores de productos y escritores altamente excitados o frustrados acuden a dichos foros. Es muy probable que veamos una afirmación perturbadora y persuasiva que está lejos de la verdad o al menos no es universalmente aplicable. En el campo de la preparación GRE, la puntuación más alta y la elección universitaria, la personalización lo es todo. Las decisiones, la enseñanza y la formación deben ser bastante matizadas y personalizadas para cada alumno. Esto tiende a faltar en un foro general, no curado.
    Los foros son un gran lugar para que interactúes con otros aspirantes a GRE como tú. Puede aclarar dudas y obtener información sobre experiencias personales, como cosas a tener en cuenta el día de la prueba, etc.

    Dicho esto, es necesario reconocer que en estos foros, son las experiencias personales, no la guía experta, las que toman la delantera. Lo que funciona para un estudiante puede no funcionar para otro y seguir ciegamente un plan sin asegurarse de que se adhiera a sus puntos fuertes y áreas problemáticas es una preocupación.
    Preste atención al premio cuando se esté preparando, es decir, obtenga la mejor admisión para su perfil y elija el método correcto de preparación hecho a medida para usted.

    GRE tiene tres componentes principales, a saber, el razonamiento verbal, el razonamiento cuantitativo y la escritura analítica. Todo el proceso de preparación debe centrarse en adquirir buenas habilidades en estas áreas y dominar los conceptos requeridos.

    Algunas de las cosas que puede hacer para obtener la claridad de concepto requerida son

    Razonamiento cuantitativo:

    La mejor parte de GRE Math es la ausencia de cosas que la mayoría de nosotros odiamos en Math – El cálculo integral, la trigonometría y otras cosas complicadas. Las matemáticas GRE son principalmente cosas que aprendió en la escuela secundaria / preparatoria: cosas como las propiedades de los entrestros, los problemas de palabras (tasa de trabajo, velocidad, probabilidad, etc.) que se pueden resolver con el sentido común básico. No mucho más allá de eso.

    Lo que hace que el GRE sea tan complicado es la interpretación de la parte de las preguntas. Una vez que descubras lo que está preguntando la pregunta, las matemáticas involucradas son definitivamente más fáciles que las que enfrentaste en tu escuela secundaria. De hecho, es más divertido ya que no es necesario anotar cada paso y mostrar a su maestro cómo llegó su respuesta. Solo necesitas elegir una respuesta. En resumen, la sección de matemáticas solo prueba tu habilidad para razonar con números.

    Razonamiento verbal:

    La sección de razonamiento verbal es aún más simple, ya que solo hay 3 tipos de preguntas: finalización del texto, razonamiento crítico y comprensión de lectura. El proceso de preparación debe estar completamente enfocado en conocer estrategias específicas para resolver estas preguntas en lugar de una preparación genérica defendida.

    Por ejemplo, el uso de tarjetas didácticas para aprender nuevas palabras fue muy útil en el formato anterior de la prueba, donde se solicitaron antónimos y analogías y se hizo más hincapié en el aprendizaje de las palabras de memoria. Pero, el formato actual se enfoca más en comprender no solo el significado literal de las palabras, sino también el uso metafórico de la palabra en función del contexto. Por ejemplo, si bien la palabra “opaco” significa literalmente “no transparente”, también podría significar “difícil de interpretar o entender” si se usa en el contexto correcto. Del mismo modo, no hay preguntas que le pidan que identifique los errores gramaticales. La gramática es importante solo para la sección de redacción de ensayos. Por lo tanto, tiene sentido no confiar en estos métodos anticuados defendidos por los institutos de coaching, sino simplemente seguir centrándose en los 3 tipos de preguntas.

    Escritura analítica:

    La sección de escritura analítica le pide que escriba 2 ensayos, uno sobre “El problema” y el otro sobre “El argumento”. La principal diferencia aquí es que en el primer ensayo se le pide que tome una posición sobre un tema complejo, mientras que en el segundo ensayo se le pide que analice la lógica detrás de una posición o posición ya tomada.

    ¿Cómo me preparo para esto? Solo debe seguir una rutina simple y tiene la garantía de obtener una buena calificación en esta sección.

    Elegir – Estructura – Escribir – Comparar – Mejorar – Elegir

    Paso 1: elige un tema en el que ya tiene algunos ensayos de muestra bien escritos. Puede recogerlos en el sitio oficial o en los muchos blogs disponibles en línea.

    Paso 2: Crea una estructura y un contorno.

    Paso 3: Escribe un ensayo desarrollando la estructura y el esquema.

    Paso 4: Compáralo con otros ensayos escritos sobre el tema.

    Paso 5: Reescribe tu ensayo para mejorarlo.

    Paso 6: Ve al Paso 1.

    ——————–

    Si haces todo lo anterior, definitivamente obtendrás una buena puntuación en tu GRE.

    Echa un vistazo al programa GRE de Magnus Prep si estás buscando excelentes mentores para que te ayuden con tu preparación GRE.

    ¡Aclamaciones!

    Me senté para GRE el 1 de agosto de 2011, el primer día del nuevo patrón. Logré 332/340 en la sección cuantitativa + verbal y 5.5 / 6 en la sección de escritura

    Tienes un año para preparar, pero yo solo me preparé durante dos días. Vaya a la sección “Cómo prepararse” a continuación y trabaje en sus debilidades. Obtener un puntaje extraordinariamente alto es una ventaja, pero por lo general, algo por encima de 315 debería ser más que suficiente para ingresar a una buena escuela, aunque no apliqué para ninguno. Vaya al centro de examen preparado con los nombres de las 5 escuelas a las que desea que le envíen su calificación de forma gratuita.

    Aquí hay un artículo que escribí como editor para el periódico del campus en IIT Kharagpur. Esto le dará una idea clara sobre el examen. He pegado el texto del artículo justo debajo del enlace. El artículo es largo, pero te recomiendo que lo leas si tomas en serio el examen.


    El Examen de registro de graduados, comúnmente abreviado como GRE, es un examen estandarizado que se debe tomar para ingresar a muchas escuelas de posgrado en los Estados Unidos y otros países de habla inglesa. El Examen General GRE, diseñado por Educational Testing Service (ETS) para la prueba de escritura. Las habilidades verbales y cuantitativas tienen como objetivo evaluar a los estudiantes independientemente de su especialización de pregrado.

    A partir del 1 de agosto de 2011, el GRE se ha renovado con la intención de hacerlo más amigable para los estudiantes. Se eliminaron las preguntas de vocabulario directo. Se proporciona una calculadora en pantalla para la sección Razonamiento cuantitativo. Como oferta promocional, a los examinados se les ofrece un descuento del 50% en la tarifa (aproximadamente Rs. 8500) para los meses de agosto y septiembre.

    Estructura del examen :
    El GRE consta de 3 tipos de conjuntos de preguntas. El examen comienza con escritura analítica, y las secciones cuantitativas y verbales (ver cuadro) siguen en orden aleatorio. Además de las secciones mencionadas, la prueba contendrá una sección experimental disfrazada de una sección cuantitativa / verbal normal. Su desempeño en esta sección no afectará su puntaje, y está destinado a ayudar a ETS a crear mejores pruebas. También puede recibir una sección de Investigación al final de la prueba, que contendrá instrucciones específicas. Se proporciona un descanso de 10 minutos al final de la sección 3, lo que hace que la prueba tenga una duración de casi 4 horas.

    ¿Cómo se evalúa?

    Sus piezas escritas son evaluadas por lectores expertos de GRE contratados por ETS. El rendimiento general de la escritura analítica se informa en una escala de 0 a 6, en incrementos de medio punto. Ambas tareas se marcarán por separado en una escala de 0 a 6 (en incrementos de un solo punto), y las puntuaciones se promediarán. Los parámetros clave incluyen la profundidad y la calidad de su argumento; Flujo lógico y uso de estructuras gramaticales correctas y efectivas. Los errores tipográficos menores, siempre que sean pocos en número, no restarán valor a su puntaje general.

    Las puntuaciones cuantitativas y verbales para el examen general revisado se informarán en una escala de 130-170 en incrementos de un solo punto. Esto significa que incluso si obtiene todas las preguntas incorrectas, su puntaje es de 260, con un máximo de 340. Después de que todas las preguntas hayan sido contestadas, su puntaje bruto (basado en el número de preguntas que intenta realizar con éxito) se normaliza al tomar Ten en cuenta la dificultad de las preguntas y el día del examen. No hay marcas negativas para preguntas individuales.

    La nueva escala de puntuación GRE aún no está lista, y las calificaciones se informarán a partir de mediados de noviembre para los estudiantes que tomen el examen en agosto y septiembre. Sin embargo, al final de la prueba se muestra un rango de puntaje tentativo en la escala anterior (200-800 para cada sección).

    Si bien el antiguo GRE fue una prueba adaptativa en computadora (responder correctamente a una pregunta significaba obtener una más difícil la próxima), el GRE revisado es una prueba de múltiples etapas, lo que significa que todas las preguntas dentro de una sección en particular tienen el mismo peso. Sin embargo, la dificultad de la siguiente sección Verbal / Cuantitativa depende de su desempeño en la primera.

    La herramienta Mark and Review

    El GRE revisado, a diferencia de la versión anterior, le permite volver a una pregunta, siempre que tenga tiempo restante para esa sección. El botón “Marcar” en la parte superior de la interfaz se puede usar para resaltar una pregunta particularmente difícil que desea intentar / revisar más adelante. La pantalla de Revisión, a la que se puede acceder en cualquier momento, le permite navegar rápidamente a cualquier pregunta (las preguntas marcadas están indicadas). Una vez que pasa de una sección, no puede volver a ella.

    Cómo preparar

    Como la mayoría de las personas consideran que las secciones de Razonamiento Cuantitativo del GRE son relativamente fáciles, la preparación para el examen generalmente se asocia con el aprendizaje de memoria de las 3.500 palabras enumeradas por el famoso Barron’s. Sin embargo, el nuevo patrón asegura que el robo de palabras abiertamente difíciles no es necesario ni suficiente. El nivel de dificultad de las pruebas de vocabulario directo ha disminuido significativamente, y el enfoque ahora está en su comprensión de las sutilezas de la construcción de la oración. Si bien es importante conocer los significados de las llamadas palabras de ‘alta frecuencia’ y otras palabras de uso común, asegúrese de que también entienda las palabras en sus contextos adecuados.

    Las preguntas de comprensión de lectura ponen a prueba su capacidad para analizar el significado del texto y para comprender el propósito de las herramientas lingüísticas empleadas por el autor. La lectura de artículos centrados en los problemas en publicaciones importantes como Wall Street Journal, Time y Frontline debería ayudar a desarrollar las habilidades necesarias para abordar estas preguntas. Asegúrese de desarrollar el hábito de leer críticamente: trate de mantener un registro de los puntos principales que se están haciendo, y cómo se utilizan los ejemplos y la gramática para hacer un punto. En conjunto con la práctica regular, este
    También debe mejorar la eficacia de su escritura.

    La práctica es muy importante: tome las pruebas de práctica a tiempo para familiarizarse con la interfaz y los tipos de preguntas. Hay bastantes libros que ofrecen preguntas de práctica y consejos generales para el GRE revisado. Los fabricantes de pruebas, ETS, han publicado su propia guía, al igual que Kaplan y Barron’s. También se puede encontrar en línea una variedad de material gratuito, en sitios web como The Princeton Review y GREEdge.

    Normas

    No hacer

    • No se distraiga con preguntas fáciles, especialmente en la sección QR. Es posible que les siga uno complicado y puede que simplemente pases por alto tu error.
    • No se preocupe por aprender de memoria los significados de palabras excesivamente difíciles. El desafío está en entender el uso de una frase / palabra.

    Dos

    • Use el descanso de 10 minutos para refrescarse, tal vez tome un refrigerio rápido. Intentar las tres primeras secciones sucesivamente puede ser bastante agotador, y desea que se le actualice y se mantenga alerta durante los próximos 90-100 minutos.
    • Para la sección de Escritura analítica, concéntrese en mantener su lenguaje limpio y su lógica lúcida. No se requieren palabras floridas y oraciones complicadas.

    Fuente: El GRE revisado: lo que usted necesita saber, de verdad.

    Copiado y copiado de mi respuesta a ¿Cuál es la mejor manera de prepararse para un examen competitivo (CAT / GRE / GMAT) en 1 año sin pagar ninguna matrícula?

    Dejame saber si necesitas mas ayuda. 🙂

    Como ya ha señalado, se siente lo suficientemente cómodo con la cantidad que nos permite enfocarnos en la parte verbal.
    Necesitará desglosar la sección verbal en los tipos de preguntas que se hacen en GRE.

    1 Vocab : Obtienes 2-3 series de secciones verbales, dependiendo de cómo venga tu sección experimental. Sin embargo, con la tendencia reciente, hay muchas posibilidades de ser verbal como experimental. Ahora que llegamos al punto, cada sección tendrá 20 preguntas, de las cuales 10 se basan generalmente en Vocab. La primera pregunta consiste en elegir la palabra apropiada y encajarla en la oración dada. Después de eso, hay 4 preguntas en las que nuevamente se rellenará la oración de tipo blan y tendrá que seleccionar 2/6 opciones que encajen en el espacio en blanco.

    Entonces esto comprende alrededor de 20/40 = 50% de tu parte verbal. La clave para abordar las preguntas de vocabulario es tratar de comprender el uso, tono y estado de ánimo de las palabras. En la mitad de los casos, sería posible eliminar las elecciones al juzgar el uso positivo / negativo de la palabra.
    Para vocab, recomendaría la aplicación Magoosh Fashcard. Apenas tardaría 15 días en terminarlo. Consiste en 1000 palabras que supongo son más que suficientes para vocab. Mirando el momento en que estás en la mano, ve también por el libro de 1100 palabras de Barrons. Para estar más seguro, también debe centrarse en las 333 palabras de uso frecuente.

    2. Comprensión de lectura: hay una gran cantidad de material disponible en RC’s. Puede comenzar con la guía oficial y luego buscar los PDF para los RC en la red. La práctica es la clave aquí. Tendrá que centrarse en 2 cosas importantes para los RC:
    1. Prueba y resuelve pasajes con una amplia gama de temas. Lo que generalmente se observa es que todos se sienten cómodos con pocos temas y se angustian mucho al ver un pasaje sobre un tema desconocido. Por ejemplo. Si usted es un ingeniero, los pasajes relacionados con la tecnología pueden resultarle fáciles, mientras que puede sentirse incómodo con un pasaje que describe alguna forma de arte antiguo. Entonces, la clave aquí es comenzar a leer pasajes con una gama más amplia de temas.
    2. Hay muchos trucos y engaños disponibles en la red si buscas en Google cómo acercarte a los RC. Mi consejo sería tratar de resolver los RC con todos y cada uno de los enfoques que puedas imaginar. Ya sea leyendo las preguntas primero, dando una lectura superficial primero, etc. Cada persona tiene su propio nivel de comodidad con cada enfoque. Tendrías que descubrir el tuyo. Tenga en cuenta el factor tiempo. Para mí, el enfoque de la fuerza bruta de leer el RC completo funcionó bastante bien. Este enfoque en su mayoría no es aconsejable, pero aún así fue bueno para mí.

    Llegando a la parte de estadísticas, cada sección contiene 2 RC (de acuerdo con la tendencia general) que van desde un total de 6-7 preguntas en cada sección. Entonces, un total de 14 preguntas de las 40, que representa el 35%.
    ¡Práctica práctica práctica!

    3. Razonamiento crítico: esta sección comprende el resto 15% de su parte verbal con 2-3 preguntas en cada sección al máximo. Los tipos de preguntas comprenden fortalecimiento / debilitamiento / suposición / negrita. Estas son las preguntas más fáciles y, con la práctica básica, puede asegurarse de no perder estas marcas del 15%. Para practicar, vuelve a empezar con el material oficial. Trate de averiguar cómo funcionan las preguntas de fortalecimiento / debilitamiento / suposición. Teniendo en cuenta el tiempo que tienes, te sugiero que sigas algunos tutoriales en you tube. Búsqueda de razonamiento crítico para GMAT. Como esta parte es muy importante y importante para GMAT, puede obtener fácilmente muchos buenos tutoriales y preguntas de práctica en you tube. Eso sería más que suficiente para la parte GRE.

    4-5 meses es una gran cantidad de tiempo. Para cubrir lo que he escrito arriba difícilmente tomará 1-1.5 meses. Una vez que haya terminado con estos conceptos básicos, le sugeriría que tome tantas simulaciones como sea posible. Ir por:
    1. ETS simulacros -2 gratis simulacros disponibles
    2. Manhattan – 6 series de prueba. Quant está un poco fuera de juego, pero sin embargo te prepararé para el objetivo de 170.
    3. Kaplan – 1 simulacro libre disponible.

    Llegar a su pregunta de que es una mejor ingesta. Definitivamente caen. Principalmente por 2 razones:
    1. El número de universidades que aceptan en otoño es casi el doble que en primavera. Por lo tanto, sus posibilidades son muy brillantes. Además, las posibilidades de obtener TA / RA son más en el otoño.
    2. Pasantías. Las pasantías son más fáciles de conseguir con el semestre de otoño.

    ¡¡Espero que esto ayude!!

    Puse 2 meses de preparación para el GRE (1 en casa durante el semestre y el segundo en la universidad) y no recomendaría prepararse para el examen durante más de 2 meses (puede que olvide cosas y oye, hay mejores cosas para pasar el tiempo en).

    Antes de la preparación, describiría mis habilidades cuantitativas como buenas (a la par con cualquier estudiante de ingeniería indio en una NIT) y mi vocabulario como deficiente. Teniendo en cuenta eso y el cronograma de 2 meses, recomendaría el siguiente plan de acción.

    Si tiene más de 2 meses antes de su examen:

    • ¡Compra este libro! Word Power Made Easy Es increíblemente barato y es un excelente manual para aprender vocabulario. Me divertí mucho haciendo sus ejercicios. Sin embargo, no aparecerán muchas palabras de aquí en el GRE real. Por lo tanto, puede omitirlo si tiene poco tiempo.

    2 meses antes de la prueba:

    • Regístrese en el sitio (ETS Home) y reserve una fecha de prueba.
    • Consigue estos 2 libros:
    1. La Guía Oficial de la Prueba General Revisada de GRE con CD-ROM, 2ª Edición (Si tiene que elegir una, consígala, ya que está a cargo de los creadores de GRE – ETS)
    2. Barron’s New GRE 2013 19th Edition

    Deberes:

    • Comencé a prepararme con Barron’s y di primero la prueba de diagnóstico. En función de su rendimiento aquí, puede descubrir las áreas en las que realmente necesita trabajar y obtener un puntaje objetivo que desee alcanzar. p.ej. Puntuaciones GRE para programas de ciencia
    • Estaba en casa para las vacaciones. Así que dediqué una hora diaria a resolver los ejercicios en Barron’s (quizás un par de horas los fines de semana).
    • Recomendaría hacer algunas preguntas de un tipo y luego cambiar a otra, en lugar de hacer todas las preguntas de práctica en un capítulo en particular a la vez. Resuelva incluso los ejemplos y preste especial atención a las explicaciones de todas las respuestas. Encontré las explicaciones para las respuestas de las pruebas de práctica especialmente útiles.
    • No hice ninguna matemática de Barron’s, ya que había descubierto que al menos para el SAT, Barron’s tiene muchos contenidos innecesarios y difíciles.
    • Ir a través de todo en el libro de ETS. Las explicaciones no son particularmente útiles, pero las preguntas son realmente buenas. Hacer todos ellos Nuevamente, no hice las preguntas de matemáticas pero sí leí la reseña de matemáticas. Recomendaría revisar el análisis de datos, las estadísticas y las partes probabilísticas correctamente, ya que algunas de estas preguntas pueden ser un poco difíciles.
    • Las matemáticas son fáciles, pero ten cuidado de no cometer errores tontos. De lo que deduje de las pruebas de práctica, necesita obtener prácticamente todas las preguntas correctas para obtener una puntuación cuantitativa perfecta.
    • Tuve algunos problemas con las preguntas de comprensión de lectura. Aquí, la sugerencia de Arjun Narayanan de tomar pequeñas notas mientras leía el pasaje realmente ayudó. Encontré que pude concentrarme más cuando hice esto.

    Vocab:

    • Recomiendo encarecidamente las tarjetas gratuitas de vocabulario GRE de Magoosh. Conseguí la aplicación de Android y comencé a perforar con ellos un mes antes de la prueba. Varias veces, el GRE usa significados secundarios de las palabras y la aplicación hace un buen trabajo de ilustrar estos significados con buenos ejemplos. Para un recurso gratuito, Magoosh es increíblemente bueno y recomendaría leer su blog GRE de vez en cuando.
    • Escribe las palabras que encuentres difíciles. Tal vez incluso hacer flashcards por su cuenta.
    • Atribuya palabras difíciles a configuraciones con las que esté familiarizado: rasgos de sus amigos, personajes de libros y programas de televisión, etc.
    • Úsalos en una conversación (los chistes que hacen tus amigos sobre tu intento de grandilocuencia pueden ayudarte a recordar el significado de una palabra mejor que cualquier otra aplicación), incluirlos en tus escritos y hacer nemotécnicas. Básicamente, ¡lo que sea para que se peguen!
    • Al igual que The New Yorker en Facebook, lea artículos sobre temas que le interesen. Cada vez que vea una palabra desconocida, intente adivinar su significado según el contexto y luego coteje con “define ” en google.
    • Un par de días antes de la prueba, reinicie su aplicación Magoosh word y vuelva a leer todas las palabras para asegurarse de que sepa más. Si olvida algunos avanzados, no se preocupe. La probabilidad de que aparezcan en la prueba es bastante baja.

    Pruebas de práctica:

    • Tanto Barron’s como la guía ETS le ofrecen 2 pruebas en papel y 2 pruebas en computadora. El GRE es un examen largo (aproximadamente 4 horas) que a veces se siente como una prueba de resistencia. Por lo tanto, definitivamente recomendaría realizar algunas pruebas de práctica antes del examen, solo para que sepa cómo mantenerse el ritmo.
    • Las pruebas en Barron’s no le dan una puntuación escalada y deben considerarse más como una práctica que como un indicador serio de cómo se desempeñaría en el GRE. Di 1 prueba basada en papel 2 semanas en y 1 más por mes. No hice las pruebas basadas en computadora ofrecidas por Barron’s.
    • Tome las pruebas de práctica de ETS mucho más en serio, especialmente las pruebas del software POWERPREP II (a las que puede acceder desde el CD que acompaña al libro o desde el sitio web a través de la descarga). Estos son muy similares a la prueba real en términos de dificultad y ofrecen la misma interfaz exacta. Asegúrese de tomar las pruebas cronometradas y simular el día de la prueba tanto como sea posible. Ir a través de las respuestas correctamente para ver lo que se equivocó.
    • Tomé las 4 pruebas ETS y alterné entre papel y computadora. Di mi último examen de práctica (basado en computadora) 5 días antes de mi GRE real.

    Evaluación de escritura analítica: puede descargar las solicitudes de AWA de los grupos de documentos e incidencias y repasarlas en su tiempo libre. Lea las respuestas de muestra y su análisis disponible en la guía de ETS para obtener una comprensión de lo que buscan.

    Asegúrate de empacar todo lo necesario el día anterior y …
    El día de la prueba:

    ¡Aclamaciones!


    Originalmente respondí aquí

    Habiendo aparecido dos veces en GRE, definitivamente puedo guiarte para estudiar para GRE.
    Es un examen muy divertido y orientado hacia el temperamento, en el que no puedes predecir tus calificaciones después de realizar algunas pruebas simuladas.

    • Si tiene un buen dominio del vocabulario en inglés, y ha leído muchos libros y novelas, el estudio durante 3 meses será suficiente para la preparación. Todavía necesitas un plan de estudio para mantenerte en el buen camino.
    • Pero, si su experiencia no incluye inglés avanzado y una capacidad de “ávido lector”, le sugiero que comience un poco antes (plan de 5 a 6 meses, como el de la escuela media vernácula).
    • Lea muchas cosas, resuelva muchos problemas de matemáticas y haga alrededor de 10 pruebas simuladas para prepararse para la última frontera.
    • Saber casi cada palabra no te dará una puntuación completa en Verbal. La sección verbal pone a prueba su capacidad para leer más rápido y comprender el significado general del pasaje o la oración.
    • No tenga la impresión de que, la sección de Quant es muy fácil y obtendré 170. Es un mito. No estoy siendo duro ni intimidante contigo, pero la sección de Quant se ha vuelto más difícil. Prepárate para algunas de las preguntas más difíciles que quizás no hayas pensado.
    • Consulte el material que es el más difícil y resuelva los libros de Manhattan. Manhattan 5lb es un libro que debe resolverse.
    • Mientras practicas, mantén un límite de tiempo y resuelve la pregunta. No se trata de rascarse la cabeza y de resolver una suma en 5-10 minutos, se trata de cuántos acertó en un tiempo estipulado. La exactitud lo es todo.
    • El autoestudio regular es necesario. Realmente depende de persona a persona, cuánto debe estudiar por día. Hay algunos planes de estudio disponibles en Magoosh, que definitivamente puedes consultar.
    • Repasar palabras. Revisar, rehacer, revisar, rehacer. Usa flashcards si quieres, más bien haz uno. No se sienta abrumado aprendiendo 100 palabras nuevas en un día. Revisalos de nuevo mañana. Es imposible recordar todas esas palabras nuevas en una sola visita. Revísalo una y otra vez.
    • Es una prueba de cuatro horas de duración, que pondrá a prueba tu fuerza mental y física. Sí, es un examen enervante.
    • La comprensión de lectura es la parte más difícil (¡Sí, la más difícil!), Practícala bien. Entiende el pasaje en lugar de encontrar la respuesta.
    • Haga sus propios conjuntos de palabras confusas que resultarán beneficiosas el día antes de su examen. No estudies nada en el día D y tampoco en la tarde del día anterior.

    Estar tranquilo y sereno el día del examen.
    Que puedas obtener la puntuación que deseas.
    Feliz preparacion 🙂

    EDITAR:
    Pensé en esto y por lo tanto requiero lo que se mencionó en la sección de comentarios.
    Gracias Pratik Mehta
    Preparar con material fácil para la sección cuantitativa es una sentencia de muerte y una pérdida de tiempo. Las personas con un buen dominio de las matemáticas tienen una calificación deficiente porque estaban muy equivocadas con el nivel de preguntas en Nova, Kaplan e incluso Barron’s, hasta cierto punto. Ninguno de estos es un buen indicador del examen final. Solo Manhattan, la guía oficial y la de Barron’s (principiantes) son útiles. Princeton también es bueno, pero no está del todo bien con el nivel de dificultad.

    El examen de registro de graduados (GRE) se usa en las escuelas de la mayoría de los países de habla inglesa para ingresar al programa de graduados. Evalúa las habilidades verbales, cuantitativas y de escritura del alumno.
    Para romper con éxito el GRE, conocer la pauta y prepararse bien es la única solución. Así que aquí hay algunos consejos para ayudarle.

    1. Construye tu resistencia mental
    El GRE dura 3 horas y 45 minutos . Tienes 1 minuto entre secciones y un descanso de 10 minutos después de tres secciones . En pruebas largas como estas, es muy probable que a la mitad de la prueba su concentración comience a disminuir y el minuto entre las secciones apenas le da el tiempo suficiente para recuperar el aliento.
    Por eso es importante desarrollar tu resistencia con suficiente preparación de antemano. Usualmente comienzas a practicar con bloques de preguntas en la misma categoría. Es fácil quedar atrapado en él, pero dominar los conceptos es solo la mitad de la batalla.
    Una vez que alcances un nivel de comodidad con las diferentes secciones individualmente, el siguiente paso será realizar exámenes de práctica completos. Programarlos en intervalos regulares durante las últimas 2 semanas antes del examen e identificar las áreas que ocupan la mayor parte de su tiempo. Estas son las áreas que probablemente te cansarán más. Controle el tiempo de cerca y trabaje para mejorar su velocidad.

    2. No subestimes la dificultad de Quant.
    El GRE está diseñado específicamente para diferir de lo que aprendiste en la universidad. Incluso si el programa de estudios de Quant te lleva de regreso a la escuela secundaria con recuerdos de las increíbles calificaciones que obtuviste, va a ser un poco más complicado que anotar en GRE. Se enfoca en probar la capacidad de razonamiento del estudiante. Así que la mayoría de las preguntas se basan en una lógica simple con un giro , convirtiéndola en un desafío para la mente. Comprender estos matices sutiles es a menudo la solución para la mayoría de los problemas.
    Intentar resolver un problema solo con conceptos y procedimientos puede ser confuso y llevar mucho tiempo. Un enfoque mucho más eficiente sería descubrir un patrón en las preguntas del truco y crear una estrategia que se pueda utilizar para ellas.
    3. Aprenda el contexto en el que se usan las palabras, no solo los significados
    Un error común cometido por los estudiantes es pasar tiempo tratando de aprender de memoria una lista de palabras dentro de un marco de tiempo limitado. Si bien las palabras son importantes para responder a las preguntas que completan las oraciones, todo el punto de las preguntas es probar el vocabulario del estudiante. De modo que conocer el significado de las palabras será inútil sin conocer el contexto en el que se usan.
    Usar ejemplos para aprender las palabras puede ser ventajoso . Cuando aprendes con ejemplos, imaginar la palabra en tu mente se vuelve fácil. Esto relaciona la palabra con una situación que a su vez hace que sea más fácil de recordar. Esto también te enseñará el contexto en el que la palabra se usa normalmente.
    Por ejemplo, una palabra comúnmente mal utilizada es literalmente .
    Literalmente significa sin exageración o en sentido estricto de la palabra.
    Entonces, cuando dices “Literalmente está lloviendo gatos y perros”, a menos que realmente vieras gatos y perros caer del cielo, estás usando la palabra “literalmente” en el sentido equivocado.
    Una palabra más apropiada en esta oración sería “figurativamente”.

    4. Prepárese bien para la Evaluación de Escritura Analítica para evitar el desperdicio de energía
    La evaluación de escritura analítica permite a las escuelas evaluar las habilidades de escritura del solicitante. Incluso si en comparación con las otras secciones, el AWA es relativamente menos importante, puede consumir una cantidad considerable de su tiempo y energía si no está preparado.
    Antes del examen, prepare un formato que describa la estructura de los 2 ensayos. Practica escribir algunos ensayos usando este formato. De esta manera, sabrás el tipo de puntos que necesitarás para el ensayo. Le permitirá enfocar sus pensamientos en términos del contenido que planea poner en el ensayo.

    5. Elige tu material de estudio con cuidado.
    Muchos estudiantes cuando se preparan eligen un libro y se apegan a él. Si bien hay muchos libros excelentes en el mercado, no has dominado el GRE si terminas uno de ellos. Incluso si comienzas con un libro, usa otro material también para complementarlo .
    El otro problema es que con la introducción de la web y el teléfono inteligente, el material de preparación disponible es abundante. Es fácil perderse tratando de hacer todo . Así que en lugar de elegir su material y prepararlo. Comience de manera simple, lo ayudará a comprender los conceptos y, una vez que haya aprendido a hacerlo, avance hacia un material más avanzado. No trates de hacer todo al mismo tiempo. Al final no habrás cubierto mucho.

    Fuente: 5 cosas que debe saber al prepararse para el GRE

    ESTO NO ES MUCHO ESTUDIO. Solo un post bastante descriptivo. Hice la mayor parte de esto en menos de una semana, pero con largas e intensas horas de estudio. Dos o tres semanas de preparación relativamente menos maníaca terminarán fácilmente todo lo que he mencionado.

    Esto no le dirá cómo se ven las preguntas GRE: todos los libros hacen eso, eche un vistazo antes de leer esto. Estas son solo algunas cosas que observé y practiqué mientras estudiaba y tomaba el examen.

    Programar:

    No recomendaría seguir el horario que hice. Fueron cinco días de estudio muy intenso. Demasiada presión y nada de diversión.

    Tres a cuatro semanas sería ideal para prepararse sin pánico. Tome una prueba de diagnóstico, que se encuentra en casi todos los libros de GRE, para evaluar lo que necesita estudiar. Es matemático e inglés básico, por lo que estudiar cosas que ya sabes es definitivamente una pérdida de tiempo. Identificar las debilidades y estudiarlas. Lo que sea que sea bueno puede ser revisado más adelante.

    Razonamiento cuantitativo:

    Así que esto es 7mo grado de matemáticas de la escuela secundaria india. No es exagerado, pero uno podría haber olvidado ciertas cosas. Así que una revisión rápida podría ayudar. Se cubren estadísticas, probabilidad, geometría, aritmética y álgebra. Sin trigonometría o cálculo, pero la trigonometría extremadamente básica a veces ayuda con las preguntas del triángulo, aunque no está prescrita para estudiar.

    La mejor manera de hacer esto sería hacer varias secciones de práctica y luego revisar lo que no sabes.

    Las preguntas de interpretación de datos a veces son confusas y requieren práctica, aunque solo sea para ver cómo se ven. Hay varios tipos de gráficos que se pueden dar y su interpretación puede requerir diferentes métodos. Estas preguntas a veces llevan más tiempo que otras, por lo que siempre las dejo para resolver las últimas.

    Las preguntas de comparación cuantitativa a menudo tienen casos de esquina que no puede considerar si no los ha practicado lo suficiente. Hay dos cantidades dadas y deben ser comparadas. Así que haga muchas de estas preguntas para comprender qué tipo de valores pueden usarse para probar las cantidades. Generalmente importantes son usar valores como 1, -1, 0, fracciones negativas y positivas aparte de otros números reales / enteros.

    Esto es cierto del GRE en general, pero más aún en cuanto a cuant. ¡No asumas nada! Las figuras NO están dibujadas a escala. Algo que parece un ángulo recto NO es un ángulo recto a menos que esté marcado o especificado.

    El resto de la cuantía es matemática básica, asegúrese de que todos los temas estén actualizados. No debería tomar mucho tiempo, si eras bueno en matemáticas en la escuela secundaria. Si se ha olvidado mucho, practica! Quant es definitivamente la sección GRE más fácil para un gran porcentaje de examinados.

    Lo que la mayoría de la gente se da cuenta es que las secciones cuantitativas son extremadamente fáciles, pero también es muy fácil cometer (increíblemente) errores tontos. Las secciones cuantitativas son de 35 minutos, 20 preguntas. La mayoría de las preguntas no tardan más de un minuto en resolverse. Intenté resolver todas las preguntas en un plazo de 20 minutos y usar el resto para resolver TODAS las preguntas nuevamente. Cambié un par de respuestas incorrectas en mi GRE real al hacer esto, así que es bastante importante REVISAR cada respuesta al menos una vez. No compruebe pasivamente. Verifique un problema resolviendo como si fuera la primera vez que lo ve, también en las pruebas de práctica.

    Razonamiento verbal:

    Las preguntas de razonamiento verbal son texto completo, equivalencia de oración o comprensión de lectura.

    No describiré cómo se ven estas preguntas, sino lo que estudié y practiqué.

    Comprensión de lectura (RC): ninguna de mis pruebas de práctica (Barrons, ETS) tuvo RC tan difíciles como las que vi en el GRE. Las preguntas de opción múltiple tienen opciones de respuestas confusas y confusas, y con frecuencia, dos respuestas parecen igualmente probables. En mi opinión, la única forma de superar esto es haciendo algunos RCs * difíciles * y buscando explicaciones de respuestas, entendiendo por qué es más probable que una opción sea la correcta. No hice NINGUNA comprensión de la práctica que fuera tan difícil como la prueba real, pero ETS Powerprep se acerca bastante.

    Otra cosa con los RC es que algunos de ellos son largos, más de 6 párrafos. Es importante saber cómo manejar estos pasajes sin perder el rastro del contenido al leer, así que practique estas preguntas.

    NUNCA asuma nada en los RCs. Todo lo que necesitas está en el párrafo. No están pidiendo su opinión o suposiciones. Si se afirma algo en una opción de respuesta, pero no está implícito en el párrafo, es incorrecto, no importa lo lógico / correcto / apropiado que le parezca. El mundo SOLO es la materia de lectura respectiva y no el mundo que te rodea. Tiene que ser implícito o mencionado por el pasaje.

    Al completar el texto y la equivalencia de la oración, los espacios en blanco deben completarse con las palabras apropiadas. Esta es una de las partes más difíciles del GRE porque las opciones de respuesta son palabras que pueden no estar presentes en un vocabulario promedio.

    Mejorar el vocabulario se hace mejor leyendo un montón. Pero el tiempo de preparación de GRE puede no ser suficiente para leer mucho, por lo que se pueden usar otras fuentes.

    Aparte de en realidad * saber * una palabra, el contexto y la eliminación de otras opciones es MUY útil. En la equivalencia de la oración, tienes que seleccionar dos opciones equivalentes. Si una opción no tiene un significado similar o equivalente a cualquier otra opción (EN EL CONTEXTO), elimínela. Lea la oración / texto a completar y comprenda el contexto antes de seleccionar las opciones.

    En el GRE que escribí, todas las palabras, excepto UNA, se encontraron en las siguientes fuentes:
    1. Mi vocabulario pre-GRE
    2. Word Power Made Easy de Norman Lewis
    3. Las 1100 palabras de Barron que necesitas saber
    4. Magoosh listas de palabras básicas y listas de palabras comunes
    5. Las 333 palabras de uso frecuente de Barron.

    Cuando tomé la prueba de diagnóstico, identifiqué el vocabulario como mi debilidad y concentré la mayor parte de los esfuerzos en esa sección (por lo que mi vocabulario inicial no fue espectacular).
    Cubrí las otras fuentes en su totalidad. No puede tardar mucho tiempo si puede hacer 300 palabras por día, pero eso es bastante doloroso. Si tiene más de dos semanas, haga 20-50 palabras por día y siga revisando. ESCRIBA las palabras que no conoces con sus significados. Es mucho más fácil de revisar desde su propia escritura a mano que desde la impresión. No puedo enfatizar eso lo suficiente. Revisa todas las palabras hechas anteriormente todos los días.

    El aprendizaje de las palabras de memoria no ayudará. El ETS está probando tu habilidad de inglés y NO tu habilidad de aprendizaje de definiciones. Por eso es muy importante aprender palabras en contexto. El 1100 de Barron hace exactamente esto y lo recomiendo a todos. Las palabras se dan en una historia (divertida) y con frecuencia recordé varias palabras debido a las historias en que las aprendí.

    Word Power Made Easy también es brillante. Se tarda un poco en leer, pero sigue así. Enseña las raíces de las palabras, así que descifrar las palabras desconocidas se vuelve fácil. Además de las raíces, también se enseñan varias palabras GRE comunes, en un formato que está estructurado. De nuevo, no hay pares de definición de palabras. Es interesante con muchos ejemplos chistosos.

    ¡Las tarjetas de Magoosh son GRANDES en camino! Android y iOS tienen una aplicación. Los usuarios de WinPhone (¡como yo!) Pueden usar cualquier aplicación de terceros que extraiga tarjetas flash de Quizlet.
    Ellos * son * pares de definición de palabras y no son suficientes para aprender el contexto correctamente. Pero los usé simplemente porque siempre tenía mi teléfono cerca. Cualquier quince minutos gratis se gastó revisando algunas palabras. Las listas básicas y comunes de Magoosh son obligatorias. No toqué las palabras avanzadas en absoluto.
    Flashcards en: chloemoreno | Quizlet
    También en el sitio web de Magoosh.

    Las 333 palabras de uso frecuente de Barron son una revisión rápida de las palabras GRE supuestamente comunes.

    El vocabulario no es extremadamente desafiante. Estas son palabras usadas en conversaciones cotidianas de alto nivel. No vi más de dos o tres palabras avanzadas en la prueba real. De nuevo, todas las palabras excepto una fueron de las fuentes estudiadas. ¡Conozca las palabras básicas antes de pasar a avanzado en cualquier lista de palabras!

    Las listas de palabras alfabéticas son confusas y bastante dolorosas, en mi opinión. Pero si puedes soportar con ellos, ve por ello.

    Ojalá hubiera tenido tiempo de hacer árboles de sinónimos. Trate de hacerlo: cómo ordenar los árboles de sinónimos de Vocab (parte 1).

    Las 800 palabras esenciales de Barron también son útiles, he oído. No tuve tiempo de pasar por eso.

    Para el conjunto verbal, hacer secciones de práctica cronometrada. Vea dónde se está perdiendo el tiempo y trate de reducirlo. Con opciones de respuestas confusas, se pierden varios minutos en el debate con uno mismo, trate de no hacerlo. Marque la pregunta (el software permite marcar) y vuelva a ella más tarde.

    30 minutos, 20 preguntas, un minuto por pregunta sería genial. Siempre hice preguntas de texto / oración primero (ya que eran rápidas) después de lo cual hice RC en cada vez más materia de lectura, simplemente porque no quería pasar el tiempo inicial leyendo largos pasajes. Encuentra un patrón en tus secciones de práctica que funcione bien para ti y síguelo.

    AWA:

    Se supone que AWA no es muy importante para los programas de ingeniería (no sé hasta qué punto esto es cierto). Sin embargo, UN ensayo de práctica de cada tipo (Asunto, Argumentativo) es una necesidad. TODOS los temas posibles ya están en Internet, publicados por ETS en sí mismos; consulte el sitio web oficial. Desafortunadamente, hay alrededor de 200 para cada tipo de ensayo, por lo que practicarlos todos es casi imposible. Escribir en 30 minutos no es fácil, así que practica al menos una vez.

    Ensayo de emisión: lea ensayos de muestra (de ETS e Internet) y descubra qué patrón de párrafos desea utilizar.

    El ensayo esencialmente le pide que analice un problema, tome una posición y apoye con ejemplos. El stand NO tiene que ser a favor o en contra del problema. Se puede inclinar hacia un lado o incluso ambivalente, si puede justificarlo.

    Utilicé la introducción, un breve resumen de mi opinión y ejemplo 1, ejemplo 2, ejemplo 3, contradicción y justificación, conclusión – 6 párrafos.

    La contradicción es un párrafo en el que te acercas a la posición opuesta y justificas tu posición con esa vista, como discutir con una opinión completamente opuesta.

    Ensayo de argumento – leer muestras (de ETS e Internet).

    Además, definitivamente busque los tipos de falacias lógicas comunes a los temas de ensayo de argumentos GRE. No hay muchos (quizás cinco o seis) y son fácilmente identificables. Si puede detectar tres o cuatro falacias lógicas, está listo para escribir el ensayo.

    Patrón que utilicé – introducción, falacia 1, 2, 3, recomendaciones, conclusión – 6 párrafos.

    GUIA DE ENSAYOS GRE
    Las 5 estrategias principales para el ensayo argumental GRE
    Blog de preparación de exámenes de Barron – Educación en línea y Preparación de exámenes

    Se prefieren los ensayos largos para ambos temas – mire las muestras de ETS. 5-6 párrafos es suficiente.

    Lo primero de los ensayos: NO están probando qué tan bueno como escritor creativo eres. NO intentes usarlos para expresar tu originalidad.

    Están probando la articulación, la gramática perfecta y la estructura lógica; por lo tanto, utilice un lenguaje sencillo en la medida de lo posible y con oraciones fáciles de entender.

    Tenga una estructura de oraciones relativamente rígida para su ensayo: luego puede dedicar más tiempo a encontrar puntos para escribir y menos tiempo a pensar en cómo comenzar el ensayo y concluir.

    Pase los primeros minutos pensando en lo que va a escribir antes de comenzar a escribir; termine con el tiempo que le queda y dedique unos minutos a revisar lo que escribe. Practique escribir con un límite de tiempo de 20 minutos, está bien si se excede un poco.

    Por cierto, no son importantes para los programas de EM, pero los programas de doctorado analizan las puntuaciones de AWA muy seriamente.

    Otro:

    Powerprep II, el software ETS, disponible de forma gratuita en su sitio web en: //www.ets.org/gre/revised_….
    DEBE HACER. El software es exactamente lo que utiliza la prueba GRE de computadora. El estándar de preguntas es muy similar también.

    Asegúrese de que todas las preguntas en el libro de texto de ETS se realicen también: fácil, medio y difícil. El estándar es como el GRE. Ejemplos de ensayos de este libro son una lectura obligada.

    LAS SECCIONES DE PRÁCTICA POR TIEMPO son muy importantes. El GRE es igualmente de velocidad y precisión.

    También tomé algunas secciones de la práctica de Barron solo para tener más exposición a diferentes preguntas.

    He escuchado que las pruebas prácticas de Manhattan son de alta dificultad. Se pueden usar si te sientes cómodo con todo lo demás. Aunque no los usé.

    Lea las páginas de ayuda de Quora respuestas / blogs / Facebook para obtener recomendaciones sobre otros recursos de práctica. Solo hice prácticas de sección cronometrada ya que no tenía mucho tiempo. Pero para familiarizarse con las preguntas, hay varios bancos de preguntas enormes (Manhattan 5 lb, por ejemplo) disponibles.

    Bueno, he dado GRE y he estudiado durante unos 20-25 días. Obtuve un puntaje de 310 (Q-160 V-150). Sé que estas marcas no son lo suficientemente buenas, pero si obtienes una ventaja inicial de aproximadamente 3 meses y te metes en el programa, fácilmente puedes anotar alrededor de 320+. Esto es lo que te recomiendo que hagas.

    1. NO UNIRSE A NINGUNA CLASE DE COACHING. Esto es lo más importante que tienes que hacer. Porque primero que nada, estás a punto de graduarte o lo has completado y asumo que tienes un buen dominio de las matemáticas básicas. Ir a clases solo desperdiciará su tiempo y, lo que es más importante, desperdiciará una buena cantidad de 20-30k en las cosas que ya sabe.
    2. Ahora, a partir de Quants, el primer trabajo importante que tendrá es repasar las matemáticas que ya ha estudiado en su grado X y XII. Esto lo puedes hacer resolviendo la BIBLIA DE MATEMÁTICAS NOVA. La BIBLIA DE MATEMÁTICAS DE NOVA (NMB) lo familiarizará con todos los conceptos básicos de probabilidad y todos los demás conceptos importantes y lo hará sentir muy cómodo con las preguntas. Una vez que haya resuelto el NMB (tomará alrededor de 2 a 3 semanas), continúe con la serie de Manhattan. Encontrará una serie de Manhattan con 8 libros en total, Manhattan 5 libras y otros 7 libros básicos de Manhattan. No empieces a resolver Manhattan 5lb todavía. Resuelve los otros libros de manhattan. Una vez hecho esto, será muy familiar para el tipo de preguntas que se hacen. Finalmente pasar a Manhattan 5lb. Ese libro será tu todo para Quants. ¡Come con ella, duerme con ella! Resuelva el libro completo (termínelo en aproximadamente un mes). También resuelve las preguntas avanzadas que se hacen. Una vez que resuelva Manhattan 5lb, tendrá una especie de dominio sobre GRE Quants. Después de eso, puede seguir adelante y resolver la GUÍA ETS (tardará aproximadamente 3 días). RECUERDE QUE LO MÁS IMPORTANTE ES MANTENER UN REGISTRO DE ERRORES. ESCRIBA TODOS LOS ERRORES QUE USTED, CUALQUIER ERROR SILLY, ESCRIBA EL REGISTRO DE ERRORES. ANTES DE UNA SEMANA O DOS DEL EXAMEN REAL, ESTÁ SOLAMENTE SU REGISTRO DE ERRORES Y ELIMINE TODAS LAS POSIBLES PROBABILIDADES DE SILLY MISTAKE EN SU EXAMEN. están terminados (NOVA, MANHATTAN, ETS), creo que se pueden obtener muy buenas calificaciones en Quants. El puntaje absoluto (170) es definitivamente posible.
    3. Pasando a VERBALES. Lo más importante de los verbales es que debe comprender el contexto de cada oración y las palabras que debe usar en ellas. Primero tendrás que mejorar tu vocabulario. Las palabras PRINCETON HIT PARADE, MANHATTAN 600 y Magoosh son más que suficientes. Si haces todo esto, puedes aprender aproximadamente 1500 palabras (la mayoría de las palabras son comunes en estos tres, pero sin embargo, haz los tres). No solo aprenda estas palabras, sino que trate de usar estas palabras en el día para usar su inglés. Haz alarde de si quieres. Lo más importante para tener éxito en VERBALS es ser muy bueno en el contexto, leer y pensar literalmente. Me refiero a los pasajes de GRE que presumen del inglés más alto y aburrido. Así que te sugiero que empieces a leer periódicos estadounidenses como The New York Times, etc. Ahora para practicar verbales, puedes hacer pasajes de Manhattan 5lb / Kaplan Series / Princeton Series. La sección verbal se basa en lo bueno que eres en inglés y no solo en inglés hablado, sino en el literal literalmente correcto en inglés.
    4. Llegando a la parte final, la sección AWA. La sección de AWA depende de su aplicación de conocimientos y de su inglés y de cómo estructurar sus puntos. La ETS GUIDE es lo suficientemente buena, ya que puede averiguar la estructura de los ensayos y también contiene un conjunto de 100-200 ensayos a los que puede hacer referencia.
    5. Ahora llegando a la parte final. ¡LOS EXÁMENES! No importa cuánto resuelva o practique, todo depende de cómo se desempeñe en el examen. Ahora, para familiarizarse con la prueba real, tiene que resolver tantas pruebas como pueda. Partiendo de la serie de pruebas de Kaplan, pase a la serie de pruebas de Manhattan. Dé estas pruebas como si estuviera escribiendo su GRE. Intente y realice estas pruebas bajo la supervisión de sus padres o cualquier amigo. Estas pruebas pueden darle una idea de dónde se encuentra y dónde debe mejorar. Realice estas pruebas de manera ocasional mientras resuelve el material que le dije antes. Ahora, aproximadamente una semana después de su examen, presente la serie ETS POWERPREP. Dar ambas pruebas con un espacio de 3 a 4 días. Estas pruebas le mostrarán dónde se encuentra exactamente y será más o menos su puntuación más alta (+ – 3)
    6. Personalmente, sugiero que en lugar de comprar libros de bolsillo, usted compre o descargue copias digitales de estos libros, ya que va a dar sus exámenes en la pantalla de la computadora, así podría comenzar a estudiar desde la misma pantalla de la computadora. Puede comprar estos libros en Amazon o los que no pueden pagarlos pueden descargar un torrent llamado “GRE PACKAGE” en los torrents kickass.

    ¡Espero que esto te ayude! Todo lo mejor

    Hay una gran cantidad de buenas respuestas aquí. Solo agregando el enfoque que tomé para los preparativos GRE.

    Antecedentes: estudiante indio de ICSE, relativamente bien leído, lector rápido
    Entrada de tiempo: valor de 4 fines de semana

    Libros y practica

    Estos son los únicos libros que he usado:

    Absoluto debe

    • La Guía Oficial de la Prueba General Revisada del GRE

    Práctica verbal

    • Preguntas oficiales de práctica de razonamiento verbal GRE
    • Libro de Problemas de Práctica GRE de 5 lb. – Manhattan Prep

    Practica cuantitativa

    • Preguntas oficiales de práctica de razonamiento cuantitativo GRE

    Pruebas de practica

    • Prueba oficial de GRE: 2 pruebas de software basadas en papel + 2 de preparación de energía
    • Pruebas diagnósticas de manhattan

    Ir a través del libro oficial de ETS GRE primero. Lea y comprenda adecuadamente las diferentes secciones y tipos de preguntas. Haga todas las preguntas de práctica para cada sección. Lea las explicaciones, donde puede aprender nuevos enfoques y consejos para resolver estas preguntas.

    Razonamiento verbal

    Esto necesita dos cosas:

    Vocabulario
    Afortunadamente, el formato GRE actual ya no requiere un vasto conocimiento de palabras oscuras. Mi lista de preparación constaba de palabras de la lista de palabras GRE de alta frecuencia 800 de @ Barron’s y @ The Ubiquitous 400.

    • Aprende palabras en contexto. Sólo las palabras y su significado se deslizarán rápidamente de tu mente. Usé Vocabulary.com para aprender las nuevas palabras. Es un sitio impresionante con definiciones bien escritas, a veces divertidas y muchos ejemplos del uso en el mundo real.
    • Las tarjetas de memoria es el método sugerido para aprender palabras, pero no es el adecuado para mi estilo de aprendizaje. En cambio, creé un archivo de texto con todas las palabras que quería aprender. Luego repasé las palabras en varios pases, cada vez que marcaba palabras que no sabía (poniendo un espacio delante de ellas y clasificando la lista al final, así obtengo todas las palabras desconocidas en la parte superior).
    • Entonces, en el paso 1, obtendría todas las palabras que no sabía o no estaba seguro de la respuesta. Luego creé una lista de palabras personalizada en Vocabulary.com Lists y practiqué hasta que aprendí todas las palabras.
    • Luego, el paso 2 se enfocó en cuál de las palabras recordé del paso 1 (nuevamente, colocando espacios adicionales en las palabras y clasificando al final).
    • Repetido hasta que tengo todas las palabras. Cada 2 horas aproximadamente, revisé el pase> = 2 palabras nuevamente.

    Práctica
    Es más importante practicar los tipos de preguntas del vocabulario GRE que solo juntar palabras. Estas preguntas verifican sus habilidades de lectura, así como la capacidad de pensamiento lógico.

    • Practica tanto como puedas. No se desanime si sigue recibiendo respuestas incorrectas inicialmente. Las respuestas incorrectas serán principalmente por dos razones: palabras desconocidas y pensamiento lógico inadecuado: no leer / entender las preguntas, etc.
    • Para palabras desconocidas: sigue anotando las nuevas palabras que encuentres y revisa cada media hora aproximadamente.
    • Por lógica impropia: fíjate bien en dónde te equivocaste al pensar. Esto es muy importante y lo ayudará a evitar el mismo error en futuras preguntas.
    • Mantuve un contador para cada tipo de error como: no sabía las palabras, no leía la pregunta correctamente, no lo razonaba hasta el final, no analizaba todas las opciones, etc. Me ayudó a hacer un seguimiento de lo que tipo de errores que estaba cometiendo, y evitaré los irritantes y tontos errores la próxima vez.

    Razonamiento cuantitativo

    Como se menciona en muchos lugares. El programa de esta sección es muy fácil para un estudiante indio. Sin embargo, seguí cometiendo pequeños y tontos errores: cálculos erróneos, sin pensarlo, etc. Por lo tanto, aquí también se requiere práctica diligente.

    • Usé solo los recursos oficiales de ETS y las pruebas de práctica para esta sección. Mantuve un registro de todos los errores que estaba cometiendo (cálculo incorrecto, no pensé en todas las posibilidades, tomé un atajo perezoso / incorrecto, no leí la pregunta correctamente, etc.) e intenté evitarlos en las preguntas futuras.
    • Ir a través de la sección de revisión de matemáticas. Sección de análisis de datos por lo general necesita una revisión, esp. percentiles y cuartiles.
    • Encontrarás muchos consejos en las explicaciones oficiales. Por ejemplo, la sección de comparación puede no necesitar cálculos completos para encontrar la respuesta.

    Escritura analítica

    No practicaba esta sección en absoluto. La noche anterior al examen, acabo de revisar los ejemplos de libros oficiales y consejos. Además, buscó ejemplos tanto para las secciones en línea.
    Por supuesto, le sugeriré que practique 2-3 preguntas de cada sección (con un límite de tiempo de 30 minutos) para formarse una idea de la estructura y el flujo de su enfoque de estas preguntas.

    Consejos generales

    • No te apresures a través de los ejercicios. Es mejor hacer 50 preguntas / palabras correctamente y recordarlas todas más tarde, que aprender 100 y recordarlas en fragmentos.
    • Luego, acelere a medida que se acerca la fecha de la prueba. En la prueba GRE real, pude completar las secciones verbales con tiempo suficiente para volver a revisar todas las preguntas, e hice un par de correcciones en las que había malinterpretado la pregunta. De manera similar para la sección de matemáticas, podría revisar la mayoría de las secciones.

    Un mes de práctica diligente debería ser suficiente para alcanzar el GRE. ¡Todo lo mejor!

    En primer lugar, el examen GRE no es tan difícil si se lo aborda sistemáticamente 320+ se puede calificar fácilmente. Aunque la puntuación en el rango de 330+ es definitivamente una tarea difícil.

    Comencé repasando el patrón del examen de la Guía oficial para el examen general revisado de GRE, y este es definitivamente el mejor y debe reservarse para GRE. Como tenía poca confianza en cuanto a Quant, comencé con la práctica de secciones verbales individuales del mismo libro. Inicialmente, cuando comienzas a practicar verbalmente, la mayoría de las palabras que encontrarás no sabrán el significado. Lo mejor es instalar un diccionario en su teléfono y ver rápidamente el significado de las palabras. Mientras practica, es esencial (especialmente para la sección RC) que se centre en el patrón de preguntas que hacen, qué proceso de pensamiento están esperando, dónde y por qué se equivoca, en lugar de centrarse principalmente en las palabras nuevas que viene. a través de. Es posible que deba practicar RC más rigurosamente que todas las demás secciones, ya que es la más difícil de todas. Después de que terminé el libro oficial, tomé prestada una cuenta de magoosh de uno de mis amigos y practiqué verbalmente desde allí (generalmente son más difíciles de lo que obtendrás en el examen real). También para hacer un mejor control del idioma inglés, debes leer mucho mientras te preparas. Preferí especialmente a las editoriales del New York Times. Me ayudo mucho. Ahora, para las palabras, solo hice palabras básicas y comunes desde la aplicación Magoosh . Si son de memoria, estará familiarizado con más del 90% de las palabras que encuentre en el examen. Si no, básicamente no debes dar el examen.

    Para la sección cuantitativa, nuevamente el libro oficial cubre todo el tema. Familiarízate con todos los temas y eres bueno para ir. La sección cuantitativa no es muy difícil, las preguntas no son muy complicadas, pero aún hay muchas posibilidades de que cometas uno o dos errores tontos.

    Guardé los AWA para el final. Básicamente la semana pasada. Puedes practicarlos escribiendo una o dos veces, eso debería ser suficiente. También lea muchas respuestas de muestra, eso ayuda mucho.

    En cuanto a las pruebas simuladas, hice todas las pruebas que obtuvimos con los libros oficiales y las pruebas gratuitas que obtuvimos con los kaplans. También asegúrese de que mientras realiza el examen, está haciendo una sesión completa que lo ayudará a aumentar la resistencia de su asiento, ya que podría ser un problema si no está acostumbrado.

    En lugar de dictar un plan para todos, he reunido los diversos elementos de preparación en los que hay que centrarse. Dado que todos tienen diferentes horarios, tiene sentido que cada persona haga su propio plan.

    Así que entremos en ello.

    1. Pruebas de practica

    Utilice la serie de exámenes pagados Manhattan Prep para su práctica de prueba simulada, son un desafío. La segunda sección de Quant te hará luchar por el tiempo, no te preocupes por eso. Es bueno pasar por una experiencia estresante antes de la prueba real, por lo que está tranquilo en caso de que ocurra el día de la prueba.

    Hay 6 pruebas disponibles aquí. Junto con esto, encontrará 1 prueba de Kaplan gratuita, 2 pruebas en la guía oficial de ETS y 2 pruebas Powerprep II gratuitas que puede descargar del sitio web de ETS. No hagas más de 1 simulacro a la semana. Y guarda las Powerpreps para el final. (Evita Crunchprep, al menos lo verbal)

    El GRE es un examen largo y largo, e incluso después de hacer muchas burlas, el último día de prueba puede ser bastante agotador. Es por eso que no tomar atajos en los simulacros, es extremadamente importante. También es importante asegurarse de realizar las pruebas aproximadamente en el momento en que dará el GRE, de modo que su mente y su cuerpo estén listos para estar alerta, a las 8 am, a las 10 am, o al espacio que vaya a reservar.

    Comience su preparación con el primer simulacro de Manhattan para saber dónde se encuentra y en qué necesita trabajar: las debilidades. Eso es gratis

    Después de eso, usando la misma identificación de correo electrónico con la que creas tu Manhattan a / c, entra en Amazon y compra el libro Manhattan Sentour Equivalence – versión Kindle. Luego envíe su recibo de Amazon a Manhattan desde la misma ID de correo electrónico. Activarán sus pruebas simuladas – 6, válidas por 6 meses. (Google este proceso una vez para asegurarse de que sigue siendo válido, no debería haber cambiado desde que lo hice hace 1.5 meses). De esta manera usted puede obtener las pruebas por menos de 500 rupias.

    2. vocabulario

    El vocabulario requiere práctica desde el principio, todos los días. Para eso, la aplicación Magoosh es buena, pero una mejor manera de hacerlo es descargar la aplicación ‘Quizlet’ en su teléfono y buscar Rajeshwar1247 o “Magoosh GRE Flashcards”. Este amable individuo ha subido todo desde el libro Magoosh Vocab a esta aplicación. Puedes aprender y hacerte preguntas a través de esto, por lo que es un método mucho más rápido. Comience a hacer esto lo antes posible.

    No te molestes en aprender demasiadas palabras, estas 1000 palabras deberían ser suficientes. La dificultad en la prueba real no es que las palabras no son familiares, sino que las oraciones son complejas.

    Entonces, en lugar de comprar más libros de vocabulario y gastar dinero, practique todas las preguntas sobre Equivalencia de oraciones y Completación de texto en el libro de Manhattan de 5lb, anote cada palabra que no conozca y haga una lista por separado. También revisa las respuestas para atrapar tus errores.

    3. Escritura analítica

    Todos los temas / temas de discusión están en el sitio web de ETS, pero revisarlos no ayuda. Primero, Cualquier guía (Guía oficial de ETS Verbal, guía de Manhattan RC) le proporcionará una explicación de los patrones generales de cada tipo de ensayo. Es importante entender esto y luego leer algunos ejemplos de ensayos con puntajes altos (nuevamente encontrados en las guías).

    Las secciones de escritura no están probando el conocimiento, sino la estructura y la capacidad de pensamiento crítico. Si tienes talento para escribir, esto no debería ser demasiado desafiante. Sin embargo, si no es así, tome algunos temas y escriba sus propios ensayos, y luego compárelos con ejemplos de ensayos proporcionados en la guía verbal o de Manhattan. Notarás que te has perdido algunos argumentos, y eso te ayudará lentamente a pensar más críticamente y escribir mejores ensayos.

    4. Práctica verbal y cuántica.

    El paso 1 es hacer la cubierta de la guía de prueba general del ETS para cubrir, para comprender la prueba y las secciones verbales y cuantitativas. Haga todas las preguntas en el libro y asegúrese de tener en claro todos los conceptos de Quant.

    Entonces empiezas a practicar. Aquí el Manhattan 5lb. El libro es el mejor recurso, con más de 2000 preguntas. Resuelva las preguntas en grupos pequeños de 20–25 y luego repase las respuestas, comprenda sus errores y note sus áreas débiles. Utilice los conocimientos de esta práctica y sus pruebas simuladas para comprender los temas en los que necesita ser más exhaustivo.

    Luego, tome el libro de Manhattan que es específico para su área de debilidad y profundice en ese tema en particular para aclarar completamente el concepto. Hice esto con desigualdades, pero no era necesario para la mayoría de los otros temas. No hagas esto a menos que sientas que es necesario.

    Para VERBAL en particular, también haga la Guía verbal oficial de ETS unos días antes de su examen GRE real, ya que es importante acostumbrarse a la forma de prueba de ETS.

    Si crea un plan que cubre 25–40 palabras de vocabulario y 20 preguntas de práctica al día, y 2 ensayos y 1 prueba de práctica simulada a la semana, estará más que preparado para un 330+

    Como beneficio adicional, hice todo esto sin gastar un centavo (excepto en las pruebas de Manhattan). Este enlace de unidad tiene todos los recursos que he mencionado anteriormente

    https://drive.google.com/drive/f

    Los encontrarás fácilmente con algunas búsquedas.

    Todo lo mejor para tu prueba y espero que esto sea útil!

    Supongo que ya es hora de que contribuya a este hilo, considerando cuánto he ganado de todas las respuestas aquí.

    Mi decisión de tomar el GRE fue hecha rápidamente. Tuve que reservar las fechas y comenzar la preparación lo antes posible. La fecha del examen fue de aproximadamente 50 días a partir del día en que me senté por primera vez con los libros; utilicé unos 45 días a partir de los 50 para mi preparación.

    Mi primera semana la pasé en investigación, y estas fueron las preguntas a las que estaba tratando de encontrar respuestas:

    • ¿Cuál es el formato de prueba GRE – tiempo, preguntas, secciones, etc.
    • ¿Cuáles son los temas a estudiar para Quant y los tipos de preguntas para verbales?
    • ¿Cuánto enfoque hay que poner para aprender nuevas palabras?
    • ¿Cuáles son las mejores fuentes de preparación tanto para Quant como para Verbal?
    • ¿Qué exámenes de práctica debo dar, y cuándo?

    También me hice la prueba diagnóstica de Barron. Su predicción fue un puntaje en el rango 315-321.

    Personalmente sentí que alrededor de 2 meses, con 2-3 horas de preparación al día son suficientes para obtener una puntuación de 315+ si provienen de un buen historial de matemáticas e inglés. De lo contrario, 4-5 meses deberían ser suficientes, pero podría estar equivocado.

    El resto de mi respuesta se basa en las respuestas a las preguntas anteriores y en cómo realicé mi preparación.

    Cuant
    Como se describe en muchas respuestas, esto es realmente matemática de la escuela intermedia. Obviamente, habrías estado un poco oxidado con eso después de todos estos años, pero no debería tomar mucho esfuerzo para devolver el brillo.

    De acuerdo, primero, ve y compra la guía oficial de ETS para el GRE. Es una necesidad absoluta y no leerlo te hará daño en algún momento. Revisé la sección de revisión cuantitativa que describía los tipos de preguntas y los temas. Yo había sido bueno en matemáticas desde la escuela (¡joder la modestia!), Y me di cuenta de que esto no sería difícil aunque con un poco de práctica.

    Leí cada palabra de Math Review del libro, y eso fue suficiente. Siento que no necesita ninguna otra fuente para aprender / revisar los conceptos y fórmulas. El lenguaje es simple y todo se explica de manera lúcida. También reconocí mis áreas débiles (permutaciones, probabilidad) después de practicar los ejercicios al final de cada capítulo.

    Después de esto, pude obtener la Biblia GRE (PDF) de Nova de un amigo. Dividí el número de páginas en el libro por el número de días que quería terminar y empecé a completar ese número de páginas cada día. La calidad de la pregunta es excelente y muchas de ellas son mucho más difíciles de lo que verías en el GRE. Resolver todos los problemas en este libro lo familiarizará con todos los conceptos requeridos y también le dará un gran impulso de confianza.

    Unos 30 días antes del examen, compré el libro de 5 libras de Manhattan. Hay MUCHAS preguntas allí, y quería hacerlas todas. Mismo enfoque – número de preguntas por número de días. La calidad de la pregunta aquí es nuevamente excelente (algunas de ellas son MUCHO más difíciles que en el GRE) y las explicaciones son fantásticas. Resolver las preguntas en este libro también lo hace pensar de la manera correcta y considerar todos los casos posibles para una pregunta.

    Resolver los dos anteriores fue más que suficiente. Me sentí bastante confiado una vez que completé las preguntas en 5 libras y se mostró en las pruebas de práctica (más sobre esto más adelante). Algunos consejos:

    • Practica bien las preguntas de interpretación de datos y anota el tiempo que tomas para resolver cada una. Esto es MUY importante: asegúrese de no anotar números incorrectos de las figuras, lea los gráficos de forma rápida pero precisa y no gaste más de unos pocos minutos (3-4) en un conjunto de interpretación de datos. Preste especial atención a lo que se ha hecho (estas preguntas generalmente involucran varias cantidades en una sola imagen, como edad, sexo, altura yada yada) y asegúrese de que no esté considerando ninguna otra cantidad al iniciar los cálculos.
    • La mayoría de las preguntas de comparación giran en torno a los casos de esquina: negativos, 0, infinito, etc. Así que revise las ecuaciones / declaraciones de problemas con cuidado y tenga en cuenta TODOS los casos que podrían aplicarse.
    • Mantente relajado Ponerse tenso arruinará todo lo que sabía bien y, de todos modos, existe una gran probabilidad de errores tontos debido a la naturaleza del GRE y las restricciones de tiempo.

    Verbal
    Para las personas de países que no hablan inglés, esta será la barrera más grande.

    Primero, los libros. Mi investigación demostró que 3 fuentes son las mejores para las expresiones verbales (aparte de ETS): Manhattan, Barron y Kaplan. Investigaciones adicionales sobre estos tres me dijeron que Manhattan sería lo mejor (por eso decidí obtener el libro de 5 libras).
    Por supuesto, como en Quant, lo primero que hice fue revisar los capítulos de ETS en la sección verbal. Me familiarizaron con los tipos de preguntas y su nivel de dificultad.

    Hay 3 partes en la sección Verbal y las abordaré por separado:
    Comprensión lectora
    Esta fue, con mucho, la parte que más me asustaba. Pasé por las muestras RC del libro ETS y me perdí. No pude obtener la mayoría de las respuestas correctas. Como lo haría un caballero, decidí dejarlo de lado y volver para luchar contra estas preguntas después de haber practicado lo suficiente.
    El libro de 5 libras fue el ÚNICO libro desde el cual practiqué RC. Pero fue más que suficiente. Tienen dos capítulos: Comprensión de lectura y RC basado en lógica (puede buscar la diferencia entre estos en línea o en un libro). La calidad de la pregunta es FANTÁSTICA, y resolver todos estos (unos 200 de los dos capítulos combinados), al mismo tiempo que se comprende la lógica subyacente y el enfoque de cada respuesta, lo preparará para abordar la mayoría de los RC. No son tan difíciles, pero esto es lo que debes hacer:

    • Preste especial atención a por qué una respuesta particular es la correcta. Comprenda los pasos para llegar a él y el pensamiento que necesita poner.
    • Lee el primer tercio del pasaje en detalle, el siguiente tercio con menos detalle y repasa rápidamente el último. No es un sermón o una carta de amor para su amada. NO intente hacer preguntas sin leer el pasaje y NO dedique varios minutos a tratar de comprender todo el pasaje. Ambos son extremos: debe encontrar un equilibrio entre comprender el pasaje y administrar su tiempo.
    • NO asuma nada que no esté mencionado en el pasaje. No puedo enfatizar esto lo suficiente, y esto también es cierto para la cuantía. Si el pasaje dice que el sol gira alrededor de la tierra, tómalo como la verdad. No ingrese sus pensamientos, conocimientos u opiniones; limítese a lo que se menciona en el pasaje.
    • Tener una imagen mental de qué parte del pasaje habló sobre qué tema. Esto es importante porque tendría que referirse a varias partes del pasaje para algunas preguntas y nunca debería pasar por todo el asunto para cada pregunta.
    • La mayoría de las respuestas se pueden llegar por eliminación. Mire una opción de respuesta y si dice absolutamente CUALQUIER COSA que vaya en contra de lo que se dice en el pasaje, elimínelo. Por supuesto, muchas veces te quedarán con 2 o más opciones probablemente correctas. En ese caso, debe poner en práctica su lógica y la práctica que realizó.

    Ahora es la parte difícil – aprender palabras.
    *************************************************** *********************
    ETS no está probando tu conocimiento de palabras oscuras, ni están probando tu memoria de palabras. Lo que están probando es cómo se pueden poner en contexto las palabras cotidianas (y algunas no comunes) y entender el significado de una oración. No encontré más de 3-4 palabras difíciles en mi examen, y creo que ese es el caso de todos. Por supuesto, he estado leyendo desde que era joven y tengo un vocabulario superior al promedio (de nuevo, f *** modestia). Pero lo que he dicho es independientemente de eso: la mayoría de las palabras eran palabras comunes que cualquier persona que haya estudiado en una escuela de inglés medio sabrá.
    *************************************************** *********************
    Inicialmente descargué la aplicación de tarjeta flash de Magoosh para Android, pero me aburrí con ella en algún momento. Así que tomé el libro de 1100 palabras de Barron e hice alrededor de 600. Recomiendo encarecidamente este libro, especialmente porque la mayoría de las palabras se presentan en contexto y muchas de ellas son palabras que puede haber usado en el pasado y son bastante comunes, pero es posible que las haya olvidado o su significado.
    Me detuve en 600 porque para entonces el libro de 5 libras había llegado. La finalización del texto y la equivalencia de la oración contenían palabras muy oscuras, y decidí simplemente hacer todas las palabras en las preguntas de esos capítulos. Creo que el total sería alrededor de 800-900. Combinando esto con los 600 que hice, mi vocabulario y la cantidad de tiempo que tenía, decidí que tendrían que hacerlo.

    Completar texto
    Repasaría unas 20 preguntas de TC en el libro de 5 libras todos los días: primero repasé todas las palabras, busqué y entendí el significado de las palabras que no conocía y luego volví a resolver las preguntas después de haber repasado todas las preguntas. las palabras.

    Equivalencia de la oración
    La misma estrategia que para TC.

    Consejos:

    • Escribe las palabras que no sabes con su significado. Revise la lista completa todos los días.
    • En SE, hay 6 opciones para cada pregunta. Por lo general (pero no siempre), puedes encontrar 2 pares de palabras de significado similar de estos 6, y el otro no estará relacionado. De los 2 pares, decide cuál encaja mejor en la oración y selecciona las palabras de ese par. Esta estrategia funciona en la mayoría de los casos y es muy, muy útil.
    • Las preguntas de TC ponen a prueba su comprensión de la oración y el significado que obtiene de ella. Esta es la razón por la que la mayoría de las opciones de respuesta para un espacio en blanco dado sonarán similares. Tendrá que volver a leer la oración 2-3 veces (pero no más) ajustando las palabras de acuerdo con su significado para determinar cuál encaja mejor. De nuevo – practicar, practicar, practicar.

    AWA
    No hay mucha preparación. Acabo de revisar las muestras de AWA del libro de 5 libras (20 de ellas) un día antes del examen. Al final resultó que, fueron las mejores muestras y el análisis proporcionado al final de cada ensayo es muy útil.
    Preste atención a la estructura y el contenido de los ensayos proporcionados en el libro. Practica la escritura de una pareja si sientes la necesidad, especialmente si nunca has escrito antes. No tengo más consejos para AWA, ya que realmente no hice mucho por eso.

    Pruebas de practica
    Hay muchos sitios que ofrecen pruebas de práctica en línea por diferentes costos. Aquí hay un consejo: ignore TODOS, excepto 3: Magoosh, Manhattan y Kaplan.
    Tenga esto en cuenta: realmente NO hay un sitio web u organización cuyas preguntas y estructura de examen coincidan con las de ETS. Solo Magoosh, Manhattan y Kaplan se acercan y valen la pena. Compre el paquete de prueba de cualquiera de estos y haga TODAS las pruebas. Esto es importante para tener una idea del entorno real de la prueba y las restricciones de tiempo, y para obtener una estimación aproximada de su puntaje (no hice ninguno de ellos por cierto, ya que no tenía tiempo, pero iba a firmar para el de Manhattan.

    Bien, ahora aquí hay dos pruebas que son las más cercanas al GRE real: las 2 pruebas de práctica proporcionadas como parte del software PowerPrep de ETS (incluidas en el CD con el libro y también disponibles en línea de forma gratuita). El software es el mismo que el del examen real y lo familiarizará con los controles y el diseño de la interfaz de usuario.
    Sin embargo, lo más importante es que las preguntas en estas pruebas son muy, muy cercanas a las del GRE real. Estas pruebas le darán la mejor indicación de su nivel de preparación, su comodidad con el examen y su preparación mental. También son los mejores indicadores de su puntuación GRE real. Esto ha sido cierto para la mayoría de los evaluadores: su puntaje real ha estado más o menos cerca de los resultados de las pruebas de PowerPrep.
    Tome estas pruebas un par de semanas antes del examen. No los tome al principio en sí mismo, ya que si no está bien preparado, los puntajes en estas pruebas disminuirán considerablemente su confianza. Use sus resultados para comprender y trabajar en áreas en las que necesita mejoras.

    Repito, NO desperdicies tu dinero en otras pruebas o paquetes de estudio, ya que ninguno de ellos imita el GRE real y te dará una impresión errónea y sobreestimada de tu preparación.

    Aquí están mis puntuaciones:
    Prueba PowerPrep I: 323
    Prueba PowerPrep II: 321
    GRE: 327 (Quant – 165, Verbal – 162, AWA – 5.5).

    1. Cree una tabla de tiempo: Primero, establezca una tabla de tiempo para usted. ¡Pasar una hora al día será suficiente! Aparte de preparar una tabla de tiempo, asegúrese de seguirla. Utiliza esa parte del día, donde puedes dedicar una hora solo para los estudios.
    2. Trabaja en lo básico: antes de intentar simular papeles, asegúrate de que tus fundamentos sean lo suficientemente buenos. Si sus conceptos básicos no son claros, entonces resolver incluso 100 simulacros no servirá de nada. Puede utilizar los primeros meses iniciales solo para trabajar en sus conceptos básicos.
    3. Haga pruebas de práctica: una vez que sus conocimientos básicos sean lo suficientemente buenos, puede comenzar a probar pruebas de práctica. Asegúrese de resolver una prueba seccional regularmente. Después de la resolución simulada, vea qué problemas tiene mal y resuélvalos de nuevo.
    4. Práctica: recuerde que para GRE, dedicar una hora diaria a los estudios puede marcar una gran diferencia. Pero, hacer todo a un mes o una semana de los exámenes no será de ayuda. Si eres un genio, entonces sería otra historia. Pero, la mayoría de nosotros no lo somos, así que sigamos el plan de practicar diariamente.
    5. Verifique su progreso: ¡ Después de resolver las burlas, vea si ha mejorado o no! Si un problema en particular lo está molestando, entonces invierta más tiempo en él. Esta es la mejor oportunidad para que usted trabaje en sus debilidades y las convierta en su fortaleza.
    6. Haga un plan de examen: la resolución simulada le dará una breve idea del tiempo que dedica a una pregunta en particular. Sobre la base de esto, puede planificar cuánto tiempo dedicar a cada problema durante el gran día.
    7. Lea mucho: leer es la mejor manera de prepararse para cualquier examen competitivo. La lectura mejorará naturalmente su vocabulario y también puede cambiar su opinión sobre ciertas cosas. Haga un hábito de leer un periódico y anote los eventos importantes del día.
    8. Tome descansos: este puntero está especialmente diseñado para aquellos que estudian en un tramo continuo. Estudiar durante largas horas puede ser agotador, así como ponerte somnoliento. Puede tomar un descanso de 10 a 15 minutos si está estudiando durante un período de 3 horas continuas. Esto ayudará a refrescar tu mente.
    9. Evite las distracciones: si usted es ese tipo de persona que no puede quedarse sin su móvil ni siquiera por un minuto, podría seguir este puntero. Durante tu tiempo de estudio, puedes apagar tu móvil. Esto te ayudará a concentrarte solo en tus estudios durante una o dos horas.
    10. Mantenga la calma: el día del examen, asegúrese de estar tranquilo. No estudie nada nuevo en el último momento y concéntrese en lo que ha preparado durante el último año. Si ha estudiado con sinceridad, entonces está obligado a obtener el resultado deseado.

    Nota: si está buscando un instituto de entrenamiento, debe asistir a un taller de Catking. Después de eso, puede decidir si aprovechar sus servicios u optar por algunas otras clases.

    Sobre CATKing

    Brindan consejos de capacitación y preparación para los mejores programas de ingreso de mba como gmat, gre, nmat, cat (cómo descifrar un IIM), snap, cet, bank po, xat, micat, cmat en sus sucursales de Andheri, Borivali, Powai en Pune y Mumbai

    http://catking.in

    Quant – 170 Verbal- 166 AWA- todavía se espera

    Recientemente di mi GRE el 28 de septiembre de 2017. Aunque comencé mi preparación 3 meses antes del examen, no pude continuar porque también me estoy preparando para otros exámenes competitivos. También fui abatido por la varicela y la tifoidea durante un mes. En efecto, tenía poco más de un mes para prepararme para el examen. Así que me sorprendió bastante el resultado que obtuve como se puede ver arriba. Así es como me preparé para mi examen GRE.

    Material de estudio –

    • Cuenta de preparación Magoosh GRE (lo más importante)
    • Magoosh Vocabulary builder (aplicación de flashcard de palabras)
    • Manhattan prep GRE 5 lb. libro de gre practica problemas
    • Manhattan GRE Prep comprensión de lectura y ensayo
    • Guía oficial para GRE por ETS (los tres libros)
    • Varios sitios de noticias como The New Yorker, The Economist, etc. (según se indica en el blog Magoosh GRE).

    Plan de estudios –

    1. Comencé mi preparación a principios del mes de julio. Como dije antes, también estoy estudiando para otros exámenes competitivos, así que no podría dedicar mucho tiempo a GRE. Estudié por un máximo de 3 horas cada día. Tampoco estudié de acuerdo a ningún plan. Algunos días solo se construía el vocabulario mediante flashcards y leyendo artículos. Algunos días estuvo practicando problemas. Algunos días divididos entre ambos. También vi las lecciones en video de Magoosh GRE y puedo decir que fueron muy útiles.
    2. Este es un consejo para estudiantes con formación en ingeniería que se preparan para el examen GRE. No ignore la sección cuantitativa, incluso si le parece fácil. Practícalo como lo harías en la sección verbal.
    3. Después de recuperarme de mis enfermedades, tenía aproximadamente 5 semanas para prepararme para el GRE. Así que decidí poner en orden mi rutina para
    • Aumenté mi tiempo de estudio GRE a 5–6 horas por día
    • En un día, primero repasaba mis tarjetas de magoosh e intentaba aprender al menos 30 palabras nuevas por día y también revisaba algunas palabras antiguas. También hice un punto para leer y entender al menos 2 artículos de noticias por día.
    • Luego practicaría cuantitativo durante aproximadamente 1 hora. En general, usé la prueba de práctica personalizada cronometrada en la cuenta GRE de Magoosh.
    • Durante las siguientes 2 horas pasaría por practicar la sección verbal y escribir ensayos.
    • Aprendí a leer los párrafos de forma rápida y detallada. Nuevamente las lecciones en video de Magoosh fueron muy importantes en este sentido.
    • Desafortunadamente, no pude realizar ninguna prueba de práctica completa debido a mi entorno, pero recomendaría a todos que le den al menos una de ellas.
    • Mi enfoque principal fue mejorar mi vocabulario al aprender nuevas palabras, su significado y el contexto de sus usos, ya que me pareció muy útil para las preguntas de equivalencia de oraciones y para completar el texto. Nuevamente, y no puedo enfatizar lo suficiente, ver videos en Magoosh fue muy importante.
    • Practiqué y practiqué tanto como mi horario me lo permitió.

    El día del examen.

    Llegue al centro al menos 1 hora antes del comienzo de la prueba, ya que hay una gran cantidad de verificación que debe realizarse antes del examen. También lleve consigo una botella de agua. El examen GRE se realiza con mucha atención y el personal de los centros de pruebas está muy atento. Si tienes algún problema no dudes en pedir su ayuda.

    GRE es un examen muy fácil, como puede ver en mi caso, con un enfoque dirigido y concentrado que puede obtener muy buenos puntajes (330+) dentro de una cantidad relativamente corta de tiempo de preparación.

    Estar bien, estar preparado y la mejor de las suertes.